You are on page 1of 102

OB/GYN Student Study Guide

Abbreviation and Definitions


LMP: last menstrual period
PMP: previous menstrual period
EDC: estimated date of confinement
GP: gravida, para: Gravida is how many pregnancies; Para is the number of
times the uterus is emptied
TPAL: (“Tennessee Power and Light”): Term (#) (the number
of term pregnancies – twins count as 1 pregnancy!) Preterm
(#) Abortions (elective or spontaneous #) Living # (all children
counted here)
G1P1002 = Twins

CKC: cold knife conization LEEP: loop electrocautery excision procedure


BTL: bilateral tubal ligation D&C: dilation and currettage POC: products of conception
Hystero: uterus TVH: transvaginal hysterectomy TAH: transabdominal hysterectomy
LAVH: laparoscopic assisted vaginal hysterectomy TLH: total laparoscopic hysterectomy
BSO: bilateral salpingoopherectomy

Oligo: few trachelo: cervix


Hyper: too much culpo: vagina
Hypo: not enough ectomy: removal of
Meno: menses ootomy: incision
Metr: uterus ostomy: making a new opening
Rrhea: flow centesis: needle into something
Rrhagia: excess flow polymenorrhea: cycle every 20 days
PROM: premature rupture of membranes PPROM: preterm premature rupture of membranes
SVD: spontaneous vaginal delivery LTCS: low transverse cesarean section
R LTCS: repeat LTCS FAVD: forceps assisted vaginal delivery VBAC: vaginal birth after c/s
VAVD: vacuum assisted vaginal delivery VMI: viable male infant VFI: viable female infant
SAB: spontaneous abortion (miscarriage) EAB: elective abortion
IUFD: Intrauterine fetal demise

ASCUS: atypical squamous cells of undetermined significance


LGSIL: low grade squamous intra epithelial lesion
HGSIL: high grade squamous intra epithelial lesion

1st Trimester: w0 – w12 gestational age


2nd Trimester: w12 – 28
3rd Trimester: w28 – 40
Previable: less than 20 weeks; if delivered considered Abortion, not SVD
Preterm: 24-37 w
Term: 37 – 42 w
Embryo: fertilization to 8 weeks
Fetus: 8 weeks to birth
Infant: delivery to 1 year
Post Dates: > 41-42 weeks

Pregnancy and Prenatal Care

Diagnosis: home UPT: highly sensitive at the time of missed cycle (positive at 8-9 d); bHCG
rises to 100,000 by 10 weeks and levels off at10,000 at term; can get gestational sac as early as
5 weeks. At that point your bHCG should be 1500 to 2000.

Discriminatory Zone: This means that when BHCG is 1200-1500, evidence of a pregnancy
should be seen on transvaginal ultrasound. When the
-BHCG is 6000,=see on ABDOMINAL U/S
FHT: seen at ~6 weeks on US; Doppler FHT at 12 w

Gestational Age: days and weeks from LMP

Dating Age (not used except on tests!): weeks and days from fertilzation; GA 2 weeks greater
than DA

Naegle’s Rule: For EDC: LMP – 3 months + 7 days + 1 year

Ultrasound: can be 1 week off in the first trimester, 2 weeks off in the second trimester, 3 weeks
in the third trimester so… if your US differs from the EDC by LMP more than this, accept the US
dating over the LMP dating. In the first half of the first trimester, use the Crown Rump Length
(CRL) which is within 3 – 5 days of accuracy.

Doppler: can get FHT (fetal heart tones) at 12 weeks (or around 10)

Quickening: at 16 – 20 weeks (mom feels the baby move)

Signs and Sx of Pregnancy:


a. Chadwick’s Sign-blue hue of cervix
b. Goodell’s Sign – softening and cyanosis of cx at 4 weeks
c. Laddin’s Sign – softening of uterus after 6 weeks
d. Breast swelling and tenderness
e. Linea nigra
f. Palmar erythema
g. Telangiectasias
h. Nausea
i. Amenorrhea, obviously
j. Quickening

Normal Changes in Pregnancy:

1. CV –
a. CO inc by 30-50% @ max 20 – 40 weeks
b. SVR dec secondary to inc. progesterone and therefore smooth muscle relaxation
c. BP dec: systolic down 5 – 10/ diastolic down 10 – 15 until 24 weeks then
slowly returns.
2. Pulmonary:
a. TV inc 30 – 40%
b. Minute Vent inc 30 – 40%
c. TLC dec 5% secondary to elevation of diaphragm
d. PA O2 and pa O2 inc; dec pA CO2 and pa CO2
3. GI:
a. Nausea and vomiting in 70% - inc. estrogen, progesterone and HCG; resolves by
14 – 16 w
b. Reflux – dec. GE sphincter tone
c. Dec lower intestinal motility, inc water reabsorption and therefore constipation
4. Renal
a. Kidneys increase in size
b. Ureters dilate – increased risk of pyelonephritis
c. GFR inc 50% - BUN, Crt dec 25%
5. Heme
a. Plasma volume inc by 50%, RBC vol inc 20 – 30% - drop in Hct
b. WBC still nl at 10 – 20 in labor
c. Hypercoaguability
d. Inc. fibrinogen, inc factors 7 – 10, dec 11 – 13
e. Slight dec in plt, slight dec in PT/PTT
Thrombocytopenia in pregnancy
 Isolated, mild (100,000-150,000/mm3)
Gestational  Asymptomatic

 Diagnosis of exclusion

Preeclampsia with  Moderate to severe (<100,000/mm3)


severe features/  Hypertension ± headache/scotomata
HELLP syndrome
 ± ↑ Creatinine, ↑ AST & ALT
 Isolated, moderate to severe (<100,000/mm3)
Immune-mediated  Asymptomatic or mucosal bleeding/bruising
thrombocytopenia (ITP)
 Normal PT, aPTT (I= ISOLATED)

Thrombotic  Severe (<30,000/mm3)


thrombocytopenic  Neurologic symptoms (eg, confusion, seizure), fever, abdominal
purpura (TTP)
 Normal PT, aPTT

Disseminated  Moderate to severe (<100,000/mm3)


intravascular  Bleeding (eg, oozing intravenous sites) ± thrombosis
coagulopathy (DIC)
 ↑ PT, ↑ aPTT, ↓ fibrinogen
ALT = alanine aminotransferase; aPTT = activated PTT; AST = aspartate aminotransferase; HELLP = Hemolysis, Elev
& Low Platelets.

6. Endocrine
a. Inc estrogen from palcenta; dec from ovaries – low estrogen levels = fetal
death and anencephaly
b. Progesterone is produced by corpus luteum then the palcenta
c. HCG – doubles roughly every 48 hours; peaks at 10 – 12 weeks; the alpha
subunit looks like LH, FSH and TSH but the beta subunit differs
d. ↑ thyroid binding globulins
e.
7. Musculoskeletal/Derm – Spider angiomata, melasma, linea nigra, palmar erythema
a. Change in the center of gravity – low back pain.
8. Nutrition – 2000 – 2500 cal/day
 need to increase protein, calcium and iron- an iron supplement is
needed in the second trimester. 30 mg of elemental iron is
recommended
i. folate is necessary early on to prevent nueral tube defect (spina bifida) –
400 mcg per day is recommended in women without seizure meds or
previous infant with neural tube defect (4g are recommended then)
ii. 20 – 30 lb weight gain is OK, obese women do not have to gain
weight.
--Rectus abdominis diastasis is a weakening of the linea alba between the rectus
abdominis muscles that can present as a nontender abdominal bulge in pregnant or
postpartum patients.  Management is conservative with observation and reassurance.

Nutrition in pregnancy

 <18.5 kg/m2: 12.7-18 kg (28-40 lb)


 18.5-24.9 kg/m2: 11.4-15.9 kg (25-35 lb)
Weight gain*  25-29.9 kg/m2: 6.8-11.4 kg (15-25 lb)

 ≥30 kg/m2: 5-9 kg (11-20 lb)


 Daily prenatal vitamin
Supplementation
 Additional specific supplements as indicated
 Substance abuse counseling
Avoid harmful  Avoidance of fish with high mercury levels
substances
 Moderate caffeine intake
 Avoid undercooked meat, fish & eggs
Food safety  Clean raw fruits & vegetables

 Avoid unpasteurized dairy products


This weight gain is equivalent to increasing caloric intake by 350 kcal/day during
the second trimester and 450 kcal/day during the thir

Prenatal Care

First Trimester: CBC, Blood Type and Screen, RPR, Rubella, Hep B s Ag, HIV, UA/Cx, GC,
Chl, PPD, Pap Smear (without cytobrush)
 Appt q mo.
 Doppler FHT @ 10 – 12 w
 OK Drugs: Tylenol, Benadryl, Phenergan
 Routine labs q visit: FHT, Fundus height, Urine dip (prt, bld, glucose, etc), weight, BP

Second Trimester: MSAFP/Triple Screen @ 15 – 18 wks, O’Sullivan @ 24 – 28 weeks


 Quickening at 17 – 19 week
 Glucose Tolerance Test Values: OSullivan: 50 g glucose  normal: under 140; if over
then perform 100 g glucose tolerance test
 Fasting 105
 1 hour 190
 2 hours 165
 3 hours 145

Rhogam @ 28 weeks

Third Trimester: RPR, CBC, Group B Strep 35-37 weeks (if not scheduled for repeat
cesarean), cervical exam every week after 37 weeks or the onset of contractions
 Labor precautions: “Go to L&D if you have contractions every 5 minutes, if you feel a
sudden gush of fluid, if you don’t feel the baby move for 12 hours, or if you have bleeding
like a period. It’s normal to have mucus or a pink discharge in the weeks preceding your
labor.”
Preventing neonatal group B Streptococcus infection
Antenatal
 Rectovaginal culture at 36-38 weeks gestation
screening
 GBS bacteriuria or GBS urinary tract infection in
current pregnancy (regardless of treatment)
 GBS-positive rectovaginal culture in current pregnancy
 Unknown GBS status PLUS any of the following:
Indications for
o PRETERM <37 weeks gestation
intrapartum
o Intrapartum fever
prophylaxis
o Rupture of membranes for ≥18 hours (1.5
days)
 Prior infant with early-onset neonatal GBS
infection 
Intrapartum
 Intravenous penicillin
prophylaxis
GBS = group B Streptococcus.

BREAST MANAGEMENT
Galactorrhea: Prolactin and TSH

This patient's bilateral, gray nipple discharge is most consistent with galactorrhea,


which is lactation in men or nonbreastfeeding women.  Physiologic galactorrhea is
usually bilateral and guaiac negative, as in this patient; the appearance is
typically milky or clear but can also be yellow, brown, gray, or green.
Although galactorrhea is typically benign, a clinical breast examination is required to
assess for underlying malignancy.  Patients with bilateral, nonbloody nipple discharge
and no findings concerning for malignancy (eg, no palpable breast mass,
lymphadenopathy, or nipple or skin changes) can then undergo evaluation for medical
causes of galactorrhea.
The most common cause of galactorrhea is hyperprolactinemia, which can occur with
a pituitary prolactinoma, use of certain medications, hypothyroidism, pregnancy, or
chest wall/nipple stimulation (eg, surgery, trauma).  Therefore, evaluation with serum
prolactin and TSH levels is required in patients with bilateral galactorrhea.  Pituitary
imaging (usually MRI) may be needed in those with elevated prolactin and/or symptoms
of a pituitary mass (eg, vision disturbances, headaches).  Patients with a normal
evaluation can be managed with reassurance and routine follow-up

Routine Problems of Pregnancy:


Back Pain GERD Constipation
Hemorrhoids Varicose Veins Braxton Hicks
Pica (cravings) Dehydration Round ligament pain (inguinal pain, worse
on
Edema Frequency walkingTX: Tylenol, heating pad,
Maternity belt)

MSAFP: produced by placenta: goes through amniotic fluid  mom


 Inc MSAFP: neural tube defects,omphalocele,gastroschisis, mult gest, fetal death,
incorrect dates
 Dec MSAFP: Down’s, certain trisomies
 TRIPLE SCREEN: MSAFP, Estriol, BHCG- risk for defects is calculated. If it comes back
abnormal, make sure dating is accurate, then counsel patient and consider
amniocentesis.
Triple Screen Tri 21 Tri 18
MSAFP dec dec
Estriol INC dec
BHCG INC dec
 Amniocentesis can be done to get baby’s karyotype if abn US, aberrant MSAFP, Adv
Maternal Age or Family history of abnormalities
 Can do a Chorionic Villi Sampling @ 9 – 11 weeks if you need a karyotype sooner, have
inc. risk of PPROM, previable delivery, fetal injury however.

 BV: metro or clinda


o ↑ Risk of preterm birth
o ↑ Risk for acquisition of HIV, herpes simplex virus type 2,
gonorrhea, Chlamydia & Trichomonas infections

-teratomas: ovarian torsion, a twisting of the ovary around the supporting ligaments
(eg, infundibulopelvic, utero-ovarian) containing the ovarian vessels.  Torsion initially
impedes venous outflow with continued arterial inflow, leading to vascular congestion
and edema.  Persistent torsion (as in this patient) causes acute-onset, severe pelvic pain
due to ovarian ischemia and necrosis from complete ovarian vessel obstruction
(eg, decreased ovarian Doppler flow) and can be complicated by peritonitis (eg, fever,
nausea, vomiting) and an acute abdomen (eg, rebound, guarding)

udden-onset unilateral pelvic pain accompanied by nausea/vomiting is most likely due


to ovarian torsion, which represents partial or complete rotation of the ovary around the
infundibulopelvic (IP) ligament.  The IP ligament contains the ovarian vessels; rotation
acutely interrupts ovarian blood flow, causing acute-onset pain.  As ovarian ischemia
progresses, patients may develop radiating pain, ovarian edema (eg, adnexal
tenderness or fullness, with or without a palpable mass), and peritonitis (eg,
rebound/guarding, fever).

- mature cystic teratomas (also known as dermoid cysts), common in premenopausal


women, have an elevated risk due to their heterogeneous composition and variable
density that creates an unstable mass prone to rotation.  This composition is seen on
ultrasound as a partially calcified mass (eg, teeth) with multiple thin, echogenic
bands (eg, hair).  In case of torsion, treatment is prompt surgical detorsion to restore
circulation and preserve fertility with either an ovarian cystectomy (to prevent
recurrence) or oophorectomy (if necrotic).

- Serous ovarian cancers (eg, cystadenocarcinoma) appear as complex masses with


solid components

- Theca lutein cysts appear as large, bilateral cystic masses rather than a unilateral
mass.  They arise from markedly elevated β-hCG levels (eg, hydatidiform mole
***CVS at 10 to 13 weeks, either by transcervical or transabdominal access to the placenta, will provide
the earliest results in order to diagnose Down syndrome. Multiple maternal serum marker analysis (Quad
screen) may be done between 15 and 21 weeks

ALCOHOL: microcephaly, cardiac anomalies, and growth retardation.

Tetracycline may cause fetal dental anomalies and inhibition of bone growth if administered during the
second and third trimesters, and it is a potential teratogen to first-trimester fetuses. tetracyclines - severe
hepatic decompensation in the mother, especially during the third trimester.

Chloramphenicol may cause the gray baby syndrome (symptoms of which include vomiting, impaired
respiration, hypothermia, and, finally, cardiovascular collapse) in neonates who have received large doses
of the drug.

Trimethoprim-sulfamethoxazole (bactrim) should not be used in the third trimester because sulfa drugs
can cause kernicterus.

- Kernicterus, the neurologic sequelae of neonatal hyperbilirubinemia, most commonly is


caused by neonatal hemolysis (eg, ABO or Rh[D] incompatibility).  Preeclampsia can
cause hemolysis and hyperbilirubinemia in the mother, but there is minimal
transplacental bilirubin transfer to the fetus.
(Choice F)  Preeclampsia causes maternal platelet consumption and
thrombocytopenia.  It does not cause neonatal thrombocytopenia, which typically occurs
due to immune-mediated thrombocytopenia.
Gonorrhea and pregnant: Ceftriaxone

M ost studies do not indicate that tobacco use is related to an increased risk of congenital
malformations, mental retardation, or developmental delay.

 Offspring of women with epilepsy have two to three times the risk of congenital anomalies even in the
absence of anticonvulsant medications, because seizures cause a transient reduction in uterine blood

Live vaccines, such as MMR, should be given at least 1 month before pregnancy, du

If not immune to rubell: give immediately postpartum

ACOG: carrier screening for Tay-Sachs disease (carrier frequency 1/30), Canavan disease (carrier
frequency 1/40), familial dysautonomia (carrier frequency 1/32), and cystic fibrosis (carrier frequency
1/29). Carrier screening tests are also available for several diseases that are less common, such as Fanconi
anemia, Niemann-Pick disease, Bloom syndrome, and Gaucher disease.

Pregnant women: omnivous diet

Neurofibromatosis, whose occurrence is often sporadic (ie, a spontaneous mutation in 50%), is inherited as
an autosomal dominant trait once the gene is in a family.

PUBS: percutaneous umbilical blood sampling: gets fetal blood to test for degree of fetal
anemia/hydops in Rh disease, etc.

Fetal Lung Maturity:


 Lecithin/Sphingomyelin Ratio: over 2.0 indicates fetal lung maturity
 “FLM”: Flouresence Polarization: >55mg/g is mature; good for use in diabetics
 Phosphatidyl glycerol: comes back pos or neg: best for diabetics because is last
test to turn positive; hyperglycemia delays lung maturity

Type II osteogenesis imperfecta

 Autosomal dominant
Pathophysiology
 Type 1 collagen defect
 Multiple fractures
 Short femur
Ultrasound findings  Hypoplastic thoracic cavity
 Fetal growth restriction

 Intrauterine demise
Prognosis  Lethal
-Achondroplasia is a non-lethal autosomal dominant bone dysplasia that presents with
macrocephaly, frontal bossing, midface hypoplasia, genu varum, and limb shortening.

-Paget disease is a bone metabolism disorder due to defective osteoclast activity.  This
disease typically has an adult, rather than in utero, onset and presents with headaches,
hearing loss, spinal stenosis, and osteosarcoma.

-Maternal vitamin D deficiency is associated with fetal growth restriction, not with
fractures or intrauterine fetal demise.
Clinic Survival Guide Copy and put in your pocket!

Clinic note:
21 yo G2P1001 at 28 2/7 by 8 week ultrasound (always include dating criteria) complaining of inguinal pain on
walking. Denies contractions, vaginal bleeding, rupture of membranes, and has fetal movement (the cardinal
questions of obstetrics).
BP 110/68 Urine: trace protein (pregnant women usually have trace protein) neg glucose
Fundal Height(FH): (measured from the pubic symphysis to fundus- correlates within 1-2 cm unless obese) 29cm
Fetal Heart Tones (FHT): 140s (count them out on your watch in the beginning; normal 120s-160s)
Extremities: no calf tenderness
(any results of recent ultrasounds, lab work here)
A/P: 1. IUP at 28 2/7: size appropriate for dates
2. Round Ligament Pain: recommended maternity belt
3. RH Neg: Rhogam 300 mcg IM today
3. Continue PNV/ Fe, discussed preterm labor precautions
4. O Sullivan today
I.M. Student, L3
Complaints:
 Discharge  do cultures, wet prep (look for trich); mucus normal at term
 The baby doesn’t move at times  babies go through normal sleep cycles. As long as it moves
every couple of hours, that’s fine. Kick counts- lie on side and count the amount of kicks in one
hour after dinner- should be over 10.

Ectopic Pregnancy

 Most common place – ampulla of the fallopian tubes; also located in ovary, abd wall,
cervix, bowel
 Risk factors: Infx of tube, PID, IUD use, previous tubal surgery, assited reproduction
 Occur in 1/100 pregnancies
 SS: episodic lower abd pain
o Abnormal bleeding: due to inadequate progesterone support
o HCG decreased: normally, HCG doubles every other day; in ectopics it doesn’t
o Unilateral tenderness
o +/- mass
o Cullen’s sign (periumbical Hematoma)
o U/S finding- complex adenexal mass, can see sac or fetus, even

 TX: Methotrexate 50 mg/m2 if <4 cm, unruptured: follow serial HCGs 4 and 7 days
later. You want the value to drop 15% between days 4 and 7. If it doesn’t, you give
another dose of methotrexate. If the mass is > 4 cm then salpingostomy or
salpingectomy (if patient is stable, can do this laparoscopically; if not needs emergent
laparotomy)
 Arias-Stella Rxn: assn with ectopic pregnancy; endometrial change that looks like clear
cell carcinoma (but is not cancerous)
MTX contraindications: herefore, absolute contraindications to methotrexate include the
following:
 Hematologic abnormalities (eg, anemia, thrombocytopenia) and
immunosuppression (eg, HIV) due to the increased risk for bone marrow
suppression and pancytopenia
 Active pulmonary disease due to the risk for pulmonary toxicity
 Hepatic or renal disease due to decreased drug metabolism and clearance,
which increases the risk for methotrexate toxicity (eg, acute respiratory distress
syndrome, bowel ischemia)
 Breastfeeding due to the transfer to breast milk, which can cause toxic levels in
the infant

 l relative contraindications, including an adnexal mass ≥3.5 cm, the presence


of fetal cardiac activity, and a β-hCG level ≥5,000 IU/L (all of which are
associated with an increased risk of methotrexate treatment failure
Spontaneous Abortions ( <20 weeks)

 Occur in 15 – 25% of pregnancies


 60% assoc with abn chromosomes (#1 cause: Trisomy 16, #2: Monosomy X)

- Parental chromosome anomalies account for 2% to 4% of recurrent losses; therefore, karyotype


evaluation of the parents is an important part of the evaluation. The causes of losses in the second trimester
are more likely to be uterine or environmental in origin. Patients should also be screened for thyroid
function, diabetes mellitus, and collagen vascular disorders. There is also a correlation between patients
with a positive lupus anticoagulant and recurrent miscarriages.

 second-trimester losses, a hysterosalpingogram should be ordered to rule out uterine structural


abnormalities, such as bicornuate uterus, septate uterus, or unicornuate uterus. Endometrial biopsy
is performed to rule out an insufficiency of the luteal phase or evidence of chronic endometritis.
 easuring the cervical length by ultrasonography is helpful in the management of patients with
recurrent second-trimester losses caused by cervical incompetence.

 3 or more mischarrages: increased risk

 The optimal time to schedule NT measurement is between 12 and 13 weeks

 First trimester screen, which includes biochemical testing with serum markers PAPP-A and free or total β-
hCG, along with maternal age
  inevitable abortion is dependent on patient preference and hemodynamic
stability.  Surgical management via suction curettage is indicated
for symptomatic (eg, dizziness on standing) and hemodynamically
unstable (eg, hypotensive, tachycardic) patients with anemia from acute blood
loss.

iagnostic criteria for antiphospholipid-antibody syndrome


(1 clinical & 1 laboratory criterion must be met)

Vascular thrombosis
 Arterial or venous

Pregnancy morbidity
Clinical  ≥3 consecutive, unexplained fetal losses before
10th week
 ≥1 unexplained fetal losses after 10th week

 ≥1 premature births of normal neonates before


34th week due to preeclampsia, eclampsia, or
placental insufficiency
 Lupus anticoagulant
Laboratory  Anticardiolipin antibody

 Anti–beta-2 glycoprotein antibody I

Valproic: NTDs and abnormal facial features.

Radiation: e anticipated fetal exposure for chest x-ray and one film of the lower spine would be less than
1 rad. This is well below the threshold for increased fetal risk, which is generally thought to be 10 rads.
High doses of radiation in the first trimester primarily affect developing organ systems such as the heart
and limbs; in later pregnancy, the brain is more sensitive.

1. Excersize: She should only perform nonweight-bearing exercises because they minimize the risks of
maternal and fetal injuries
2. Since the physiologic changes associated with pregnancy will persist from 4 to 6 weeks following
delivery, women should not resume the intensity of pre-pregnancy exercise regimens immediately
following delivery.

3. The most common medical complaint in adulthood in patients with achondroplasia is symptomatic
spinal stenosis.

 One person has a balanced translocation


 Carriers of balanced translocations of the same chromosome are phenotypically normal. However,
in the process of gamete formation (either sperm or ova), the translocated chromosome cannot
divide, and therefore the meiosis products end up with either two copies or no copies of the
particular chromosome. In the former case, fertilization leads to trisomy of that chromosome.
Many trisomies are lethal in utero. Trisomies of chromosomes 13, 18, and 21 lead to classic
syndromes. In the latter case, a monosomy is produced, and all except for monosomy X (Turner
syndrome) are lethal in utero.

U/s

- An encephalocele is a version of a NTD that involves an outpouching of neural tissue through a defect in
the skull. A cystic hygroma, with which encephalocele can often be confused on ultrasound, emerges from
the base of the neck with an intact skull present. Hydrocephalus is related to the size of the lateral
ventricles. Anencephaly would require absence of a much larger proportion of the skull with diminished
neural tissues. An omphalocele is a defect in the abdominal wall at the insertion of the umbilical cord,
which may lead to herniation of the abdominal contents. Omphaloceles are associated with various other
birth defects and chromosomal abnormalities.

- Women who have a previously affected child have a NTD recurrence risk of about 3% to 4%.

Prior hx: 4 mg of folic acid daily

No hx: 400ug

No folate: Ultrasound shows a large defect in the calvaria and meninges, with only a
small cerebellum and brainstem, 

Ddx

 Incomplete vaccination against rubella or varicella increases the risk of congenital


infection, which can present with fetal growth restriction (rubella) or microcephaly
(varicella) on ultrasound.  Neither is associated with anencephaly.
(Choice C)  Poorly controlled hypertension, as in this patient, increases the risk of fetal
growth restriction, placental abruption, preeclampsia, and need for preterm delivery.  It
does not increase the risk of NTDs.
(Choice D)  Undercooked meat consumption is associated with congenital
toxoplasmosis, which can present with microcephaly, hydrocephalus, and diffuse
cerebral calcifications on ultrasound.  It does not cause NTDs.
(Choice E)  Some untreated sexually transmitted infections such as syphilis and herpes
simplex virus (HSV) increase the risk of congenital infection, which may present as
hydrocephalus (syphilis) or microcephaly (HSV) on ultrasound.  They are not associated
with anencephaly.

 RF if recurrent: infx, maternal anatomic defects, Antiphospholipid Sd; endocrine


problems (of mom), previous miscarriage
 LABS to do: bHCG, CBC, type and screen, US; give Rhogam if Rh -
 Definitions:
o Threatened AB – intrauterine pregnancy with bleeding; closed cervix  needs
initial obstetric visit
o Missed AB – Fetal death without passage of products of conception; no FHT by
8 weeks
o Inevitable AB – dilated cervix, proceeds to complete or incomplete
o Incomplete AB – products not all out  do a D&C
o Complete AB – Products all out; need to follow BHCG until 0 to make sure it
was not a hydatidiform mole or choriocarcinoma
 SS: bleeding, crampy abdominal pain (always ask if clot or whitish tissue was passed)
 Abortion @ 6 – 8 week: 1. Trisomies 2. Turner’s Sd (45X)
 Habitual Ab: 3 Ab’s in a row
o Causes: balanced translocation of parents, autoimmune dz, abn uterus, etc.
o WU: karyotype for balanced trans, antiphospholipid ab, hysterosalpinography for
abn uterus (septate uterus most common)
 Incompetent Cervix Sd: Ab’s between 13 – 22 weeks because cervix can’t hold POC in:
see painless dilation and effacement in 2nd trimester; infx is common b/c of
trauma/vaginal flora
TX: McDonald’s Cerclage: a pursestring nonabsorpable suture around
cervix: remove at term; also could manage expectantly; BEDREST – give steroids and
Abx to dec infx and inc fetal lung maturity and tocolyze contractions; Both McDonald and
Shirodkar are near the internal os – Shirodkar stitch just tunnels under the cervical
epithelium.
 Causes of 2nd Trimester Abs: infx, mat anat defects, cervical defects, systemic dz,
fetotoxic agents, trauma (chromosomes occur in second trimester, but not as frequently
as first trimester)

Chromosome Stuff

 Trisomies: 13 Edwards, 18 Patou, 21 Down’s


 Autosomal Dominant Dz: Neurofibromatosis, von Willebrand’s, Achondroplasia,
Osteogenesis imperfecta
 X Linked Dz: Muscular Dystrophy, G6PD Def, hemophilia
 Recessive Dz: 12 OH Adrenal hyperplasia
 McCune Albright: polyostotic fibrous dysplasia: degeneration of long bones, sexual
precocity, café au lait spots (tx precocious puberty with medroxyprogesterone acetate)

25% chance
Statistical Stuff

 Maternal Mortality = mat death/100,000 live births


 Fertility rate = # live births/1000 females 15 – 44
 Birth rate = # live births / 1000 people
Antepartum Fetal Surveillance

 NST = Non Stress Test: to be “reactive” need 2 accelerations, of 15 beats per minute
for 15 seconds in 20 minute strip; if nonreactive, baby can be sleeping – give mom
juice – do a BPP (think about sedatives, narcotics, CNS/CV abnormalities)
 BPP = biophysical profile; on U/S 8 pts good/ 4 pts bad

Give 2 points Give 0 points


NST Reactive < 2 accels
AFI (amniotic Fluid Index) one 2 by 2 cm pocket no pocket seen
Fetal Breathing Movements Last over 30 seconds < 30 seconds
Fetal Extremity Movements 3 or more episodes Under 3 episodes
Fetal Tone Extension to flexion; flex at rest Extended at rest

 Modified BPP = NST and AFI


 Contraction stress test (CST): nipple stimulation or oxytocin – shows 3 uterine
contactions in 10 minutes to be good; negative = no late decelerations
 HOW TO READ THE STRIP:
o Reassuring things – normal behavior, beat to beat variation, reactive strip
(above)
o Early decels – they begin and end with the contraction – a sign of head
compression – OK
o Variable decels – are more jagged and look like a V – a sign of cord compression
– we may start amnioinfusion
o Late decels – begin at peak of contraction and end after contaction is finished – a
sign of uteroplacental insufficiency – are bad. (nonreassuring)
 FSE = fetal scalp electrode- placed usually with IUPC when a more accurate recording of
heart tones is needed; do not use in moms with HIV
 IUPC = Intra Uterine Pressure Catheter – placed in uterus to monitor contractions; a good
baseline is 10-15 mm Hg; Ctx in labor inc. 20 – 30 mmHg or even to 40 – 60; can
amnioinfuse through the IUPC with normal saline- You cannot tell how strong a
contraction is with the tocometer. You need an IUPC to count MonteVideoUnits.Over 200
MVUs is considered adequate.
 Fetal Scalp pH; take blood from scalp for nonreassuring factors, fetal hypoxia (not really
done anymore)
PH over 7.25 is reassuring 7.2 – 7.25 indeterminate <7.2 bad
Tuberous Sclerosis
There are a few periungual fibromas on the hands and fee
Tuberous sclerosis complex

 Mutation (inherited or de novo) in TSC1 or TSC2 gene


Pathophysiology
 Autosomal dominant
 Dermatologic
o Ash-leaf spots
o Angiofibromas of the malar region
o Shagreen patches
 Neurologic
Clinical features o CNS lesions (eg, subependymal tumors)
o Epilepsy (eg, infantile spasms)
o Intellectual disability
o Autism & behavioral disorders (eg, hyperactivity)
 Cardiovascular: rhabdomyomas

 Renal: angiomyolipomas
 Tumor screening
o Regular skin & eye examinations
o Serial MRI of the brain & kidney
Surveillance o Baseline echocardiography & serial ECG
 Baseline electroencephalography

 Neuropsychiatric screening

IF minimal variability + no accels Do fetal scalp


stimulation test to look for accel if nothing possible
hypoxia

ITCHING

Polymorphic eruption of pregnancy

Clinical features  Pruritic, erythematous plaques in striae


 Spares periumbilical region
 Spares face, palms & soles

 Develops in third trimester


Diagnosis  Clinical

 Topical corticosteroids
Treatment
 Antihistamines
 No adverse pregnancy outcomes
Prognosis
 Resolves immediately postpartum

-Chronic inflammation results in a pruritic, erythematous papular rash that is limited


to the abdominal striae.  The PEP rash may develop a centrifugal spread but
typically spares the umbilicus, face, hands, and feet.
PEP is diagnosed clinically and is common in nulliparous women or those with multiple
gestations.  It does not require additional obstetric management as it is not associated
with adverse maternal-fetal outcomes.  Therefore, management is control of pruritic
symptoms; treatment typically includes antihistamines and topical corticosteroids. 
PEP typically resolves spontaneously 

DDX
Ursodeoxycholic acid is used in patients with intrahepatic cholestasis of pregnancy,
which typically presents with pruritus during the third trimester.  Patients typically have
pruritus worst on the hands and feet and have no associated rash.
-Antiviral agents (eg, valacyclovir) are indicated in the management of herpes zoster,
which can present with an erythematous papular rash confined to the abdomen.  Herpes
zoster typically is a unilateral rash (following a dermatomal pattern) that spreads beyond
the abdominal striae, and lesions evolve into vesicles and bullae.

Labor
DATING
 Menstrual History: 40 weeks from LMP (Naegle’s rule: LMP + 7 days – 3 months)
 Uterine Size:
o 10 Weeks grapefruit size
o 20 weeks is at umbilicus
o 20 – 33 weeks matched dates +- 2 cm of Fundal Height
o may not match at term due to descent
 Ultrasound: is most accurate at 8 – 12 weeks
 Dating Criteria for delivery: determines whether lungs are considered mature for delivery
1. FHT documented 30 weeks by Doppler.
2. 36 weeks since UPT positive.
3. US of CRL at 6-11 weeks makes gestational
age >39 weeks.
4. US of under 20 weeks supports gestational
age >39 weeks.

STAGES OF LABOR
 First: beginning of contractions to complete cervical dilation
o Latent – to approx. 4 cm (or acceleration in dilation)
o Active – to 10 cm complete; prolonged if slower than 1.2 cm/hr null/1.5 cm/h
multip; if prolonged, do amniotomy, start pitocin, place IUPC to evaluate
contraction strength
o Failure to progress – no change despite 2 hours of adequate labor (MVU >200)
 Second: complete dilation to the delivery of baby
o Prolonged if 2 hours multip/ 3 hours nullip (with epidural) or 2 hours nullip/1
hour multip (no epid)
 Third: delivery of baby to delivery of placenta
o Can take up to 30 mins
o Signs include increase in cord length, gush of blood, uterine fundal rebound
 Fourth: one hour post delivery

Disorders of the active phase of labor

Diagnosis Clinical features

 Cervical change slower than expected


Protraction
 ± Inadequate contractions
 No cervical change for ≥4 hours with adequate
contractions
Arrest OR

 No cervical change for ≥6 hours with inadequate


contractions

3 P’S OF LABOR
1. Power: nl contractions felt best at fundus; last 45-50 seconds; 3 in 10 minutes
2. Passenger:
a. Presentation – what is at the cervix (head (vertex), breech)
b. Position – OA, OP, LOT, ROT
c. Attitude – relationship of baby to itself
d. Lie – long axis of baby to long axis of mom
e. Engagement – biparietal diameter has entered the pelvic inlet
f. Station – presenting part’s relationship to ischial spine (-3, -2, -1, 0, 1, 2, 3)
3. Pelvimetry:
a. Inlet: Diagonal Conjugate – symphysis to sacral promontory = 11.5 cm
Obstetrical Conjugate – shortest diameter = 10 cm
b. Midplane: spines felt as prominent or dull
c. Outelt: Bituberous Diameter = 8.5 cm
Subpubic Angle less than 40 degrees

FORCEPS
 Outlet forceps: requirements –
 visible scalp
 Skull on pelvic floor
 Occiput Anterior or Posterior
 Fetal head on perineum : can see without separating labia
 Adequate anesthesia; bladder drained
 Maximum 45 degrees of rotation
 Low forceps:
 station 2 but skull not on pelvic floor

 Midforceps: station higher than 2 with engaged head (not done)

VACCUUM EXTRACTION: can cause cephalophematoma and lacerations


 Same requirements for outlet forceps

INDUCTION:
 Indications: PreEclampsia at term, PROM, Chorioamnionitis, fetal jeopardy/demise,
>42w, IUGR
 Bishop Scoring System: if induction is favorable: >8 vaginal delivery without induction
will happen same as if with induction: < 4 usually fail induction: < 5 – 50% fail induction

Score Cm Effacement Station Consistency Position of cx


0 0 0-30% -3 Firm Post
1 1-2 30-50% -2 Med Mid
2 3-4 60-70% -1,0 Soft Ant
3 4-5 >80% +1, +2

 Prostaglandins: dilate cervix and inc contractions: Prepidil, Cervidil, Cytotec:


contraindicated in prior CS, nonreassuring fetal monitoring
 Laminaria: an osmotic dilator, is actually seaweed!
 Amniotomy: speeds labor; beware of prolapsed cord!
 Oxytocin: 10 U in 1000 ml IV piggyback on pump @ 2 m U/min; if over 40 mU/min are
used watch for SIADH
 Augmentation of labor needed in inadequate ctx, prolonged phases
Late- & post-term pregnancy
 Late-term: ≥41 weeks gestation
Definition
 Post-term: ≥42 weeks gestation
 Prior post-term pregnancy
 Nulliparity
Risk factors  Obesity
 Age ≥35
 Fetal anomalies (eg, anencephaly)
 Fetal/neonatal
o Macrosomia
o Dysmaturity syndrome
o Oligohydramnios
o Demise
Complications
 Maternal
o Severe obstetric laceration, from big
baby
o Cesarean delivery
o Postpartum hemorrhage

 Frequent fetal monitoring (eg, nonstress test)


Management
 Delivery prior to 43 weeks gestation

-placenta accreta include prior cesarean delivery (particularly in patients with a placenta
previa in the current pregnancy) and prior uterine myomectomy.
-Polyhydramnios (single deepest pocket ≥8 cm or an amniotic fluid index of ≥24 cm) is
associated with congenital fetal malformations (eg, esophageal atresia, anencephaly)
and maternal diabetes mellitus.

-decreased placental function due to age-related placental changes (eg, infarctions,


calcifications) that cause increased placental vascular resistance.  Progressive placental
dysfunction can lead to uteroplacental insufficiency and chronic fetal hypoxemia,
which cause central nervous system suppression and intrauterine fetal demise.  To
prevent this suppression, blood is preferentially distributed to the brain rather than
peripheral tissue.  This redistribution can be evidenced on ultrasound
as oligohydramnios (single deepest pocket <2 cm or amniotic fluid index <5 cm) as
amniotic fluid is dependent on renal perfusion and urine production.
DELIVERY
 Crowning - Ritgen’s maneuver (hand pressure on perineum to flex head) Head out:,
check for nuchal cord (cord around neck) – delivery anterior shoulder gently by pulling
straight down- suction nares and mouth with bulb – deliver posterior shoulder – clamp
cord with 2 Kellys, cut with scissors, hand off baby – get cord blood– gentle traction on
cord with suprapubic pressure, massage mom’s uterus – retract placenta out and inspect
it – inspect mom for tears, visualize complete cervix
 Episiotomy repair (1 – 2 degree midline) 2 – 0 Chromic or Vicryl locking suture superiorly
to repair vaginal mucousa – interrupted chromics to repair deep fascia if needed – simple
running to repair mid fascia – sub Q stitch inferiorly and superficially
 A third degree tear involves the rectal sphincter; a fourth degree tear involves rectal
mucousa
 Midline episiotomy: can extend, but has less dyspareunia; Mediolateral episiotomy is
done at 5 or 7 o’clock, but has more pain and infx but less chance of extension (consider
if shoulder dystocia)

third-degree perineal laceration, which tears the external anal sphincter (EAS), or a


fourth-degree perineal laceration (ie, EAS and rectal mucosa tear).  Risk factors include
fetal macrosomia (as in this case), prolonged second stage of labor, and operative
vaginal delivery (eg, forceps).
Obstetric anal sphincter injuries are usually identified and repaired immediately after
delivery.  However, anal incontinence (ie, involuntary loss of fecal material and/or
flatus) may still occur in the first few weeks after delivery due to labor-related stretching
of the pudendal nerve and pelvic floor muscles (both of which normally help maintain
continence).  Therefore, patients with postpartum anal incontinence require evaluation to
determine if symptoms are due to a benign cause (eg, pelvic floor stretching) or
from breakdown of the EAS:
 In patients with a normal physical examination (eg, normal anal sphincter tone),
symptoms will likely resolve as the pelvic floor recovers.  For these patients,
reassurance and observation are appropriate.
 In contrast, patients with weakened anal sphincter tone and asymmetric
contraction (such as this patient) or a palpable defect are likely to have an anal
sphincter muscle defect that requires additional management.  For these
patients, the best next step is endoanal ultrasonography, which can locate and
assess the extent of the defect to guide treatment (eg, repeat surgical repair) and
decrease the risk of chronic anal incontinence

 Shoudler Dystocia
RF: macrosomia, DM, obese, post dates, prolonged second stage.
Compl: fracture, brachial plexus injury, hypoxia, death
Treatment:
1. Suprapubic Pressure (not fundal pressure!)
2. McRobert’s – mom flexes hips – knees to chin level
3. GENTLE traction
4. Wood’s Corkscrew – pressure behind post shoulder to dislodge the ant shoulder
5. Rubin maneuver – pressure on accessible shoulder to push it to ant chest of
fetus to decrease biacromial diameter
6. Fracture clavicle away from baby
7. try to deliver posterior arm

CARDINAL MOVEMENTS
 Engagement – fetal head enters pelvis
 Flexion – smallest diameter to pelvis
 Descent – vertex to pelvis
 Internal Rotate – sag suture is parallel to AP
 Extend at pubic symphysis
 Externally rotate after head delivery

INDICATIONS FOR C-SECTION


 Failure to progress (P’s of labor)
 Breech presentation with labor
 Shoulder presentation
 Placenta Previa
 Placental Abruption
 Fetal distress: 5 minutes of decal <90 bpm; repetitive late decals unresponsive to
resusitation
 Cord Prolapse
 Prolonged second stage of labor
 Failed forceps
 Active herpes
 Prior classical C/S (has to do with incision on uterus not skin!)
 2 prior low transverse c/s (VBACs are controversial)

AFTER C/S

ostpartum fever (≥38 C more than 24 hours after delivery), wound induration, and
incisional erythema most likely has cellulitis, a superficial surgical site infection due
to incisional bacterial contamination during her cesarean delivery.  Risk factors for
wound infection include obesity and emergency surgery (eg, inadequate skin antisepsis
or antibiotic prophylaxis).

Patients typically develop superficial surgical site infection within days to weeks after


surgery.  Typical signs include wound induration and erythema, which begins at the
incision site at the level of the dermis but may also spread to surrounding subcutaneous
tissue, resulting in edema, subcutaneous fluid accumulation, and increased peri-
incisional pain.

Treatment of nonpurulent cellulitis is with first-generation cephalosporins (eg,


cephalexin) because they cover common skin pathogens (eg, beta-hemolytic
streptococci, methicillin-susceptible Staphylococcus aureus [MSSA]).

(Choice B)  This patient's hematuria and urinary retention are likely due to bladder and
urethra trauma during labor and general anesthesia.  Urinary retention is managed with
Foley catheter replacement, which can cause catheter-introduced bacteriuria and
cystitis.  This patient's urinalysis is normal (eg, negative leukocyte esterase and nitrites),
making this diagnosis unlikely.

(Choice C)  Septic pelvic thrombophlebitis can cause postpartum fever due to injury,
thrombus, and hematogenous spread of infection to the pelvic veins (eg, ovarian veins). 
Patients are typically initially treated for suspected endometritis but have relapsing-
remitting fevers and persistent abdominal pain despite antibiotics.  Septic pelvic
thrombophlebitis is not associated with incisional induration or erythema.

(Choice D)  Breast tenderness and engorgement can be due to normal postpartum milk
letdown or signs of developing mastitis, which is caused by obstruction, inflammation,
and infection of the breast ducts.  However, patients with mastitis typically have high
fevers, malaise, and unilateral breast erythema (not seen in this patient).
(Choice E)  Patients who undergo cesarean delivery are at risk for endometritis, a
polymicrobial infection of the intrauterine cavity and common cause of postpartum fever. 
Patients with endometritis typically have uterine fundal tenderness, which is not seen in
this patient.

Educational objective:
Superficial surgical site infection (ie, cellulitis) can occur after cesarean delivery and
present with postpartum fever (≥38 C more than 24 hours after delivery) as well as
incisional induration and erythema.  Risk factors include obesity and emergency surgery
(eg, inadequate skin antisepsis or antibiotic prophylaxis).

heavy vaginal bleeding, up to 20% have minimal or no bleeding because blood is


concealed within the intrauterine cavity.  These patients typically have focal pain (eg,
back pain, suggesting posterior placental rupture), an abnormally distended uterus (eg,
measuring 38 cm at 35 weeks gestation), and high-frequency contractions due to
increasing intrauterine pressure and volume.  As more of the placenta detaches, fetal
hypoxia can occur, as evidenced by abnormalities on the fetal heart rate tracing (eg, late
decelerations).

Ultrasound

Doppler Velocimetry: systolic/diastolic ratio in the umbilical cord


 Inc S/D ratio: pre-eclampsia, IUGR, nicotine, maternal tobacco
 If end diastolic flow absent or reversed, delivery is indicated
 Velocimetry is done in cases of suspected IUGR
The first ultrasound is the only one that can change dates. Accept U/S date if over LMP date by…
 4d – 1 w: first trimester
 2w: second trimester
 3 w: third trimester
Dating is done by a biparietal diameter, head circumference, femur length and abdominal
circumference.

Anesthesia
Epidural anesthesia: lengthens second stage – may need oxytocin
 Injected into L3/L4 interspace: use the technique of least resistance (the epidural space
has a negative atmospheric pressure so the syringe you place over the needle will
suddenly lose its resistance as you advance it into the epidural space, inject test dose)
 Can cause hypotension after dosage because the autonomic nervous system is blocked
and all blood pools in extremities; can see late decals, but usually resolve with hydration
and blood pressure increase

.
Paracervical block: not really done because can inject into fetus easily and cause fetal
bradycardia
Spinal: one time dose, shorter duration of action, used in repeat c/s
Pudendal Block: Can be done with vaginal delivery, inject analgesic into post-ischial spine and
sacrospinous ligament (takes 5 – 10 mins to set up: good for forceps delivery without epidural)

Postdural puncture headache

 After lumbar puncture or neuraxial anesthesia


 Positional (worse when upright, improves when supine)

Clinical features  Neck stiffness


 Photophobia, diplopia

 Hearing loss, tinnitus


 Typically self-limited
Management
 Epidural blood patch
causing cerebrospinal fluid leakage, low cerebrospinal fluid pressure, and resultant
slight herniation of the brain and brainstem.  Therefore, patients can develop
a positional headache (ie, headache worsens when sitting or
standing and improves with lying down), which correlates with the increased and
decreased herniation/pressure on the brain, within 72 hours of the procedure.  Other
associated symptoms may include nausea, vomiting, and neck stiffness.

Ddx
Preeclampsia with severe features can develop postpartum, particularly in patients with
gestational hypertension, and present with headache and edema.  However, headaches
due to preeclampsia are not positional.  In addition, this patient is normotensive with
normal deep tendon reflexes, making this diagnosis less likely.

Fetal Complications of Pregnancy

SMALL FOR GESTATIONAL AGE


 < 10% percentile for growth
 can be symmetric or asymmetric
 has higher rates of mort/morbidity
 RF: Decreased growth potential
o Congenital abn: Tri 13, 18, 21, Turners
o CMV, Rubella
o Teratogens, smoking, EtOH

IUGR:
 Causes: Htn, DM, renal dz, malnutrition, plac previa, abruption, CMV, Toxo, Rubella and
mult gest
 Symmetric: insult was early in gestation ie. Viral
 Asymmetric: late onset (ie. Tobacco); femur length is usually spared
 Doppler velocimetry with end diastolic flow reversed or absent or nonreassuring fetal
heart tracing necessitates delivery.
Fetal growth restriction

Asymmetric (BIG HEAD,


Symmetric
small belly)

Definition  Ultrasound estimated fetal weight <10th percentile or birth


weight <3rd percentile for gestational age
Onset  1st trimester  2nd/3rd trimester

 Uteroplacental insufficiency
 Maternal malnutrition
 Chromosomal
Etiology abnormalities
 Asymmetric FGR is associated with
maternal hypertension and tobacco use
 Congenital infection
during pregnancy

 Clinical
features  Global growth lag  Head-sparing growth lag

 Regular nonstress testing


 Weekly biophysical profiles
Management  Serial umbilical artery Doppler sonography

 Serial growth ultrasounds


FIRST TRIMESTER #1 cause: chromosome, then infection

weight <10th percentile or birth weight <3rd percentile for gestational age.

MACROSOMIA: > 90% percentile: > 4500g


 Higher risk of shoulder dystocia and birth trauma (brachial plexus injuries), low APGAR,
hypoglycemia, polycythemia, hypocalcemia, jaundice
 ETIO: DM, obesity, post term, multiparity, inc. age
 FU: u/s q 2 weeks to assess size; however US is not that accurate in diagnosis
 TX: tight control of diabetes; wt loss before conception; induce, prepare for dystocia;
consider c/s if over 5000g

OLIGOHYDRAMNIOS:
 Amniotic Fluid index: divide mom’s belly into 4 quadrants – measure the largest pocket of
fluid in each <5: Oligohydramnios >20: Polyhydramnios
 Absence of Range of Motion – 40X increase in Perinatal mortality
 Assn with abnormalities of GU (renal agenesis = Potter’s Sd, polycystic kidney dz,
obstruction), and IUGR
 Fetal Kidney/lung  amniotic fluid  resorbed by placeta, swallowed by fetus, or leaked
out into vagina.
 Most common cause: ROM (rupture of membranes)
 Dx: US
 TX: If preterm, hydrate if fetus stable; If term, deliver

POLYHYDRAMNIOS:
 AFI > 20 or 25; 2-3% of pregnancies; assn with NT defects; obst mouth, hydrops, mult
gest
 Monitor with serial ultrasounds. Can do therapeutic amniocentesis.

Antenatal Hemorrhage

PLACENTA PREVIA: Abnormal implantation of placenta over the internal os


 Three types
1. Complete (completely over os)
2. Partial (little over os)
3. Marginal (barely over os)
 SS: painless vaginal bleeding – dx by ultrasound – DON’T EXAMINE WITH YOUR
HANDS ! Avoid speculum exam! If patient presents complaining of vaginal bleeding,
make sure an ultrasound for placental location is performed first.
 RF: previous placental previa, prior uterine scars, multiparous, adv mat age, large
placenta
 TX: CS if lungs mature/fetal distress/hemorrhage

 Placenta accreta: superficial invasion of placenta into wall of uterus


 Placenta increta: invasion into the myometrium
 Placenta percreta: invasion into the serosa
Tx for above 3: 2/3 get hysterectomy after c/s
-- this patient's major risk factor is her prior cesarean delivery.  Increasing cesarean
delivery rates have caused an increased incidence of placenta previa because the
uterine scar and change in vascularity likely alter early pregnancy implantation. 
Additional risk factors include prior placenta previa, multiple gestation (ie, increased
placental surface area), and advanced maternal age (age ≥35) (Choice C).  Most
cases resolve spontaneously by the third trimester due to physiologic lower uterine
segment lengthening and/or placental growth toward the fundus.  Patients with
persistent previa undergo cesarean delivery at 36-37 weeks gestation (ie, prior to the
onset of labor).

ACCRETA: the umbilical cord avulses from the placenta, necessitating manual
extraction.  The placenta is extracted in pieces, and the patient suddenly develops
profuse vaginal bleeding.  The uterus is firm, and the bleeding is unresponsive to uterine
massage and uterotonic medications.

Definition  Morbidly adherent placental attachment to the myometrium

Risk factors  Placenta previa + prior uterine surgery (eg, cesarean delivery, D&C, myomecto

 Prenatal diagnosis: US with placenta previa, numerous placental lacunae, myo


Clinical features
 Postpartum diagnosis: adherent placenta, postpartum hemorrhage
Management  Cesarean hysterectomy with placenta in situ
D&C = dilation & curettage; US = ultrasound.

- An emergency hysterectomy is typically required to minimize further maternal blood


loss and achieve hemostasis.

PLACENTAL ABRUPTION: premature separation of a normally implanted placenta


 SS: usually painful vaginal bleeding (uterus is contracting) / hemm between wall and
placenta
 RF: htn, prior abruption, trauma, smoking, drugs – cocaine, vascular disease
 DX: inspection of placenta at delivery for clots; can see retroplacental clot on ultrasound
or a drop in serial hematocrits
 TX: deliver if fetal status nonreassuring
 Complications: hypovolemia, DIC, couvalaire uterus (brown boggy), PTL

Definition  Placental detachment from the uterus before fetal delivery

 Hypertension, preeclampsia
 Abdominal trauma
Risk factors  Prior abruptio placentae 

 Cocaine & tobacco use


 Sudden-onset vaginal bleeding 
 Abdominal or back pain
Clinical presentation  High-frequency, low-intensity contractions

 Rigid, tender uterus


 Clinical
Diagnosis
 Ultrasound: ± Retroplacental hematoma 
 Fetal hypoxia, preterm birth, mortality 
Complications
 Maternal hemorrhage, disseminated intravascular coagulation
- The bleeding also increases intrauterine pressure, which causes constant pain, a rigid,
tender uterus, and uterine irritability (ie, high-frequency contractions).

Accreta : POSTPARUM HEMORRHAGE

Aspiration (chemical) pneumonitis from ↑ prog gastric aspiration

 Gastric reflux (ie, ↓ esophageal sphincter tone)


 Delayed gastric emptying
Risk factors  ↑ Intraabdominal pressure
 Altered consciousness or sedation (eg, anesthesia)

 Endotracheal intubation, nasogastric tube


Pathophysiology  Aspiration of gastric acid with direct tissue injury
 Lung parenchymal inflammation
 Within hours of aspiration event
Clinical  Acute-onset dyspnea, low-grade fever, hypoxemia
 Diffuse crackles on lung examination
presentation
 Chest x-ray infiltrates (dependent lung segments)
Management  Supportive care (ie, no antibiotics)

UTERINE RUPTURE : major cause of maternal death


 40% assn with a prior uterine scar (CS, uterine surgery)
 60% not assn with scarring but abd trauma (MVA), improper oxytocin, forceps, inc. fundal
pressure, placenta percreta, mult gest, grand multip, choriocarcinoma/molar pregnancy
 SS: severe abd pain, vag bleeding, int bleeding, fetal distress
o FETAL PARTS ABOVE THE FUNDUS!!!
 TX: immediate laparotomy, hysterectomy with cesarean

FETAL VESSEL RUPTURE: occurs usually with a velamentous cord insertion between amnion
and chorion; may pass over os=vasa previa (Perinatal mortality 50%)
 SS: vag bleeding, sinusoidal variation of HR
 RF: mult gestation (1% singleton, 10% twins, 50% triplets)

NON OBSTETRIC CAUSES OF ANTEPARTUM HEMORRHAGE


Cervictis, polyps, neoplasms, vag laceration, vag varicies, vag neoplasms, abd pelvic trauma,
congenital bleeding d/o
Preterm Labor

 RF: low SES, nonwhite, <18 yo, mult gest, h/o preterm birth, smoking, cocaine, no PNC
uterine malformation, h/o CKC, Group B strep, Chlamydia, Gonorrhea, BV
 SURVIVAL: 23 w 0-8% 24w 15-20% 25w 50-60% 26-28w 85% 29w 90%
 ALGORITHM:

Good Dates

<24w 24-34w >34w

Sab Tocoysis, Steroids Expectant management

Cervical insufficiency: FEELING OF FULLNESS in lower pelvis, cant feel cervix

 CONTRAINDICATIONS TO TOCOLYSIS: acute fetal distress, chorioamnionitis,


eclampsia/pre e, fetal demise, fetal maturity, hypersensitivity to tocolytics, heart disease,
IUGR
 WORK UP: H&P, check cervix visually by speculum, wet prep, UA, cervical length, fetal
fibronectin
 TOCOLYTICS:
 MgSO4: works as membrane stabilizer, competitive inhibition of Ca;
therapeutic at 4-7 mEq/L
 SE: flushing, nausea, lethargy, pulm edema
 Toxicity: cardiac arrest (tx: calcium gluconate), slurred speech,
loss of patellar reflex (@ 7 -10), resp problems (@15-17),
flushed/warm (@9-12), muscle paralysis (@15-17), hypotn
(@10-12)
 Nifedipine: calcium channel blocker: 10 mg q 6 h; se: nausea and
flushing
 B2 agonist: ritodrine/ terbutaline: dec. uterine stimulation; may cause
DKA in hyperglycemia, pulm edema, n/v, palpitations (avoid with h/o
cardiac disease or if vaginal bleeding) 0.25 mg sq q 20-30 min x 3 then 5
mg q 4 po
 Indomethacin/prostaglandin synthesis inhibitor: 50 mg po/100 mg pr
SE: premature closure of PDA in an hour, CAUSES oligohydramnios
 ndomethacin tocolysis (ie, to inhibit contractions) is
indicated in patients with preterm labor at <32 weeks
gestatio

 ADD…
o Betamethasone or Dexamethasone (to increase fetal lung maturity)
o Bedrest with bathroom priviledges
o Pen G (Group B Strep prophylaxis)

 PRETERM BABY RISKS


o Low birth weight
o Intraventricular hemorrhage
o Sepsis
o Necrotizing enterocolitis
 PREVENT


o

FALSE LABOR
Contraction onset, frequency, duration, and pain level can help distinguish between
labor and false labor (ie, Braxton-Hicks contractions).  Patients in labor have regular,
painful contractions (eg, palpable) that cause cervical changes (eg, dilation, effacement)
and may have associated vaginal bleeding or leakage of fluid.  In contrast, those in false
labor have mild, irregular contractions that cause no cervical change.  Fetal
monitoring is typically via nonstress test, which evaluates fetal acid-base status and risk
for fetal hypoxemia.  Those with a reactive nonstress test (eg, moderate variability,
accelerations) require no additional evaluation.
Although this preterm patient has risk factors for preterm labor (eg, advanced maternal
age, iron deficiency anemia), she has irregular contractions, a closed cervix, and a
reactive nonstress test.  Therefore, she is in false labor and can be discharged home
with labor precautions as her contractions will likely resolve without intervention.
(Choices A and E)  Patients in preterm labor have regular, painful contractions causing
cervical change; management is based on gestational age.  In patients at <32 weeks
gestation, magnesium sulfate is administered for fetal neuroprotection (eg, cerebral
palsy prevention), and indomethacin is started for tocolysis.  Indomethacin is
contraindicated after 32 weeks gestation due to the risk of premature fetal ductus
arteriosus closure.  In patients at <37 weeks gestation, betamethasone may be
administered to prevent neonatal respiratory distress syndrome.  This patient is not in
preterm labor.

Ultrasound cervical length measurements are performed in the second trimester for
patients with a prior spontaneous preterm birth.  They are not performed in the third
trimester as the cervix undergoes physiologic dilation and effacement closer to term, and
therefore measurements do not predict preterm delivery.

Arrest of diliation
econd stage of labor is the period from complete cervical dilation (10 cm) to fetal
delivery.  This patient achieved excellent fetal descent to +3 due to her average-sized
infant (eg, 3.4 kg), suitable pelvis (no fetal molding or caput, suggesting no resistance
against the bony maternal pelvis), and favorable fetal position (left occiput anterior). 
However, she meets criteria for second-stage arrest (ie, no further fetal descent), which
are:
 ≥3 hours of pushing in a primigravida without an epidural; some providers allow
additional time with an epidural (ie, ≥4 hours pushing with an epidural, as in this
patient)

OR
 ≥2 hours of pushing in a multigravida without an epidural (≥3 hours pushing with
an epidural)

Continued pushing increases delivery complications (eg, postpartum hemorrhage)


with limited chance of spontaneous vaginal delivery.  Therefore, the best next step in
management is operative vaginal delivery (eg, vacuum-assisted) to expedite delivery. 
Other indications for operative vaginal delivery include maternal exhaustion, fetal
distress, and maternal conditions in which the Valsalva maneuver is not recommended
(eg, hypertrophic cardiomyopathy).

PROM

Preterm PROM <37w (usually 32-36 w) = PPROM

Prolonged PROM : rupture > 24 hours


 CAUSES: infx, hydramnios, incompetent cervix, abruptio placenta, amniocentesis
 Labor usually follows shortly
 DX: Sterile speculum exam – ferning (on slide), pooling (in fornices), nitrazine paper
(turns blue) - gc, chl, strep B culture U/S – looks for AFI (oligohydramnios)
 MGMT: > 36w  delivery
Preterm  pen G for B strep, expectant management vs. delivery for any signs
of infection or fetal compromise, BPPs vs. NSTs

Chorioamnionitis

 Def: infection of amniotic fluid


 Requires delivery; increased risk with inc. length of rupture of membranes
 SS: fever > 38 c, inc WBC, tachycardia, uterus tender, foul discharge
 TX: Ampicillin and Gentamycin, add Clindamycin if c/s, DELIVERY
 Most common cause of neonatal sepsis
Intraamniotic infection (chorioamnionitis)

 Prolonged rupture of membranes (>18 hours)


 Preterm prelabor rupture of membranes
 Prolonged labor
Risk factors  Internal fetal/uterine monitoring devices
 Repetitive vaginal examinations

 Presence of genital tract pathogens


Maternal fever PLUS ≥1 of the following:
 Fetal tachycardia (>160/min)
Diagnosis
 Maternal leukocytosis

 Purulent amniotic fluid


 Broad-spectrum antibiotics
Management
 Delivery

Complications  Maternal: postpartum hemorrhage, endometritis


 Neonatal: preterm birth, pneumonia, encephalopath

Endometritis
 RF: prolonged labor, PROM, more c/s than vag delivery
 ORGS: polymicrobial  anerobes/aerobes like E Coli/Group B Strep/Bacteroides
 SS: uterine tenderness, foul lochia
 TX: gentamycin and clindamycin (continue until 24-48 h afebrile)

Cephalopelvic Disproportion

 Common indication for c/s


 Types of pelvis:
 Gynecoid: 12 cm widest, sidewalls straight
 Android: 12 cm diam, sidewalls convergent
 Anthropoid: <12 cm, sidewalls narrow
 Platypelloid: 12 cm, sidewalls wide
 Obstetric conjugate diameter: sacral promontory to midpoint symphysis pubis: shortest
AP diameter 9.5 – 11.5

Malpresentation
Breech: 3-4%
 RF: previous breech, uterine anomalies, polyhydramnios, oligohydramnios,
multigestation, hydro/anencephaly
 Frank: flexed hips, extended knees (feet near head)
 Complete: flexed hips, one or both knees flexed
 Incomplete/Footling: one or both foot down
 DX: Leopold’s maneuver, vaginal exam (feel sacrum and anus)
 TX: C Section is the preferred management, external version (manipulation into
vertex position), trial of delivery if 2000-3500g and multip (has a proven pelvis)

Face: chin is anterior for delivery, many anencephalics have a face presentation; dx on exam
Brow: must convert to occiput for delivery
OP: usually rotate to OA (manually)
Shoulder: transverse lie  do c section
Compound: fetal extremity with vertex or breech  cord prolapse; part will reduce as labor
occurs

--Breech presentation occurs when the buttocks or feet are the fetal part closest to the
maternal cervix.  Risk factors include prematurity, multiparity, multiple gestation, uterine
anomalies (eg, septate uterus), leiomyomas, placenta previa, and some fetal anomalies
(eg, hydrocephaly).  Breech presentation is typically diagnosed via ultrasound; however,
examination findings consistent with a breech fetus include subcostal pain or a palpation
of a hard mass near the uterine fundus (due to the fetal head) or lack of a fetal
presenting part on digital cervical examination.
ry; therefore, the procedure is performed at ≥37 weeks gestation to decrease the risks
associated with premature delivery.

-However, vaginal delivery is contraindicated in patients with footling or


incomplete breech presentation (such as this patient) due to the risk of head
entrapment through an incompletely dilated cervix after delivery of the body,
which is an obstetric emergency.  Because this patient desires a vaginal delivery, ECV
should be attempted; however, if the procedure is unsuccessful, a scheduled cesarean
delivery is indicated.
External cephalic version

 Manual rotation of fetus to cephalic presentation


Procedure
 Decreases cesarean delivery rate
 Breech/transverse presentation
Indications
 ≥37 weeks gestation
 Contraindication to vaginal delivery
Absolute o Prior classical cesarean delivery
o Prior extensive uterine myomectomy
contraindications
o Placenta previa
 Abruptio placentae
Complications
 Intrauterine fetal demise

PP Hemorrhage

Defined as > 500 ml blood loss following vag delivery, > 1000 ml blood loss following c/s
 Causes
o Uterine atony coagulopathy
o Forceps uterine rupture
o Macrosomia uterine inversion
 TX
o Vigorous fundal massage Oxytocin 20 U in 1000 ml NS
o Repair laceration Methergine 0.2 mg IM (contra: htn)
o Take out placental remnants PgF2 – alpha (Hemabate) (contra: asthma)
o Cytotec 800 mg rectal Hysterectomy if medical therapy fails
-This contrasts with more extensive lacerations, which disrupt the anal sphincter muscles
(third-degree) and rectal mucosa (fourth-degree) and can lead to anal or fecal
incontinence.

Rh Incompatibility

 Mom is Rh neg (Rh is an antigen on the RBC: CDE family) + Dad is Rh pos = baby is be
Rh pos: during first pregnancy (usually at delivery but can occur with Sab,amniocentesis,
trauma, ectopic, etc), mom develops antibodies against Rh positivity (because she lacks
the antigen) which can cross the palacenta and cause a hemolysis in the newborn which
may cause death.
 Kleihauer Betke Test: assess amt of fetal blood passed into maternal circulation
 On first visit: blood type, also screen for other antibodies:
o Lewis – “lives”
o Kell – “kills”
o Duffy – “dies”

 RHOGAM: given as passive immunization to prevent sensitization: given @ 28 w; check


baby at delivery, if Rh+  give Rhogam again to mom within 72 hours
 If multip not sensitized  tx as above
 Sensitized: mom has developed antibodies against baby  check a titer: if over 1:8, do
fetal survey on US and amniocentesis at 16 – 20 w to measure the OD 450 with the
spectrophotometer (you know, that machine you used in general biology) reading for the
LILEY CURVE

Zone 3  HDN

Zone 2 

Zone 1  Okay

Weeks gestation

Note: the delta OD 450 is prognostic, not the titeritself


Zone 2/3 TX: intrauterine blood transfusion through umbilical A of RH neg blood
 ERTHROBLASTOSIS FETALIS: heart failure, diffuse edema, ascites, pericardial
effusion, bilirubin breakdown  jaundice, neurotoxic effects.
 SOMETIMES DOSE IS NOT ENOUGH: tandard dose of 300 µg at 28 weeks
gestation can usually prevent alloimmunization.  However, ~50% of Rh-negative
women will need a higher dose after delivery, placental abruption, or
procedures.  The Kleihauer-Betke (KB) test is commonly used to determine the
dose.  Red blood cells from the maternal circulation are fixed on a slide.  The
slide is exposed to an acidic solution and adult hemoglobin lyses, leaving "ghost"
cells.  The dose of anti-D immune globulin is calculated from the percentage of
remaining fetal hemoglobin.

Indications for prophylactic administration of anti-D
immunoglobulin for Rh(D)-negative patients*

 At 28-32 weeks gestation


 <72 hours after delivery of Rh(D)-positive infant
 <72 hours after spontaneous abortion
 Ectopic pregnancy
 Threatened abortion
 Hydatidiform mole
 Chorionic villus sampling, amniocentesis
 Abdominal trauma
 2nd- & 3rd-trimester bleeding

 External cephalic version 

Diagnostic test
Etiology

Male factor  Semen analysis

Ovulatory function  Midluteal phase (day 21) progesterone level

 Day 3 FSH & estradiol levels


 Clomiphene citrate challenge test
Ovarian reserve  Antral follicle count

 Antimüllerian hormone
Fallopian tube patency  Hysterosalpingogram

Uterine cavity evaluation  Sonohysterogram


Progesterone, secreted by the corpus luteum, increases body temperature 1-2 days
after ovulation.  Therefore, basal body temperature (BBT) records can identify
anovulatory patients (no monthly BBT increase) or inform timed intercourse in ovulatory
patients.  Although tracking BBT could help this ovulatory patient time intercourse, male
factor infertility should be excluded first due to its high prevalence.
-  female factor causes of infertility are investigated.  Women are assessed for normal
ovulation (eg, menstrual history, midluteal phase progesterone level), ovarian reserve
(eg, antimϋllerian hormone level), and anatomy (eg, hysterosalpingogram for fallopian
tube patency, sonohysterogram for intrauterine anomalies)
Intrauterine Fetal Demise

 IUFD assn with abruption, congenital anomalies, post dates, infection, but usually is
unexplained.
 Retained IUFD over 3 – 4 w leds to hypofibrinogenemia secondary to the release of
thromboplastic substance of decomposing fetus  sometimes DIC can result.
 DX: no FHT on ultrasound
 TX: delivery
 Postdates :@ 41 w: do NST: if nonreassuring do induction
o 42w: do BPP and NST 2 q wk: if nonreassuring do induction
o inc risk of macrosomia: oligohydramnios, Meconium aspiration, IUFD
o DX: by LMP, u/s consistent with LMP in first trimester
o Induce after 42 w

Multiple Gestation: 1/80 twins & 1/7000 – 8000 triplets


 Complications: PTL, placenta previa, cord prolapse, pp hemorrhage, pre E
 Fetal complications: preterm, congenital abnormalities, SGA, malpresentation
 Delivery: usually occurs at 36 – 37 w if twins; Triplets – 33 – 34 w

Monoygotic Twins: “identical”


1. Dichorionic diamniotic: 2 chorions/ 2 amnions: separation before trophoblast on
embryonic disk (splits before 72 hours)
2. Monochorionic diamniotic: has one placenta; when twins occur d. 5-10 before
amnion forms
3. Monochorionic monoamniotic: one chorion and amnion; can be conjoined twins

Dizygotic Twins: “fraternal”


1. Dichorionic diamniotic
2. Inc in Africa (Nigeria)
3. 2 sperm/ 2 eggs
 DX: u/s, inc HCG, inc MSAFP
 TX: managed as high risk
 Delivery of Twins:
o 40% vertex  vaginally (only if reassuring FHT, 2500 – 3500 g)
o 20% vtx / br or br / vtx 20%  controversial, usually c/s
o 20% br / br  cs
o Triplets  cs

Hepatic issues
Acute cholangitis

Etiology  Ascending infection due to biliary obstruction


 Fever, jaundice, RUQ pain (Charcot triad)
Clinical presentation
 ± Hypotension, AMS (Reynolds pentad)
 Cholestatic liver function abnormalities
o ↑ Direct bilirubin, alkaline phosphatase
Diagnosis o Mildly ↑ aminotransferases

 Biliary dilation on abdominal ultrasound or CT scan


 Antibiotic coverage of enteric bacteria
Treatment
 Biliary drainage by ERCP within 24-48 hr
AMS = altered mental status; ERCP = endoscopic retrograde cholangiopancreatography; RUQ = right upper quadrant.

-fever, right upper quadrant (RUQ) pain, and jaundice (Charcot triad) are most likely due
to acute cholangitis.  Pregnant women are at increased risk for cholesterol gallstone
formation because elevated levels of progesterone and estrogen promote gallbladder
stasis and cholesterol supersaturation.  These gallstones can become impacted in the
common bile duct, resulting in biliary obstruction and subsequent RUQ pain, jaundice,
and direct hyperbilirubinemia.  As bacteria from the small bowel enter the biliary
system, patients develop infection (eg, fever, leukocytosis) and possible sepsis (eg,
hypotension, altered mental status).

Acute fatty liver of pregnancy can present in the third trimester with RUQ pain,
jaundice, and elevated transaminases.  However, patients typically have signs of
fulminant liver failure, including thrombocytopenia (<100,000/mm ) and profound
3

hypoglycemia, not seen in this patient.

-Preeclampsia with severe features can present with RUQ pain and elevated
transaminases; however, this diagnosis requires maternal hypertension (≥140/90 mm
Hg), not seen in this patient.

Ddx
Symptomatic cholelithiasis in pregnancy

 ↑ Biliary cholesterol excretion (estrogen)


Pathophysiology
 ↓ Gallbladder motility (progesterone)
 Recurrent, postprandial epigastric/RUQ pain
Clinical features
 RUQ ultrasound with echogenic foci (stones or sludge)
 Conservative (eg, pain control)
Management
 Cholecystectomy (for complicated, recurrent cases)
RUQ = right upper quadrant.
Ddx
ver congestion can cause RUQ pain in patients with decompensated right heart failure. 
Peripartum cardiomyopathy, which primarily causes left-sided heart failure, can lead to
secondary right-sided failure.  This patient has no signs of left-sided heart failure (ie, no
pulmonary edema), making this diagnosis unlikely.  Dyspnea, a midsystolic murmur (due
to physiologic changes in flow), and 1+ pitting edema are normal in pregnancy.

-ddx
Hepatic adenomas are benign liver tumors associated with high estrogen levels (eg,
pregnancy, combination oral contraceptive use).  Ruptured hepatic adenomas can cause
RUQ pain; however, patients have acute onset of pain, a possible palpable liver mass,
and hemodynamic instability (eg, hypotension, tachycardia) due to intraabdominal
bleeding.

-Preeclampsia can cause RUQ pain due to stretching of the liver capsule; however, this
diagnosis is unlikely in patients without hypertension (ie, systolic ≥140 mm Hg or
diastolic ≥90 mm Hg).

PreEclampsia / Eclampsia / Chronic Htn

Normal Mild Pre E Severe Pre E


BP <140/90 140-159/90-109 >160/110
Dip Prt TR +1,+2 +3,+4
24h Urine <150 mg 300 mg 3.5 – 5.0 g
H/a, vision changes No no yes
RUQ pain No no yes
HELLP, LFT increased No no yes

 ETIOLOGY: vasospasm; inc. thromboxane; trophoblast invasion of spiral arteries


 recurrence of pre E in subsequent pregnancy is 25 – 33%
 Fetal Complications: prematurity, dec blood flow to placenta; abruption/fetal
distress, IUGR, oligohydramnios
 SS: htn, proteinuria in third trimester
 When severe, can get severe h/a, vision changes, seizures (eclampsia)
 RF: nulliparous, >40 yo, African American, chronic htn, chronic renal dz, antiphospholipid
sd, twin gestation, angiotensin gene T235, SLE
 TX: delivery is the “cure”
MgSO4 (always check reflexes and respirations when on Mg, need good UOP)
4.8 – 8.4 mg/ml: therapeutic
8 CNS depression
10 Loss of dtr’s
15 Respiratory depression/paralysis
17 Coma
20 Cardiac Arrest
Hydralazine to control BP over 160/110
 ECLAMPSIA: pre eclampsia plus seizures
o Can have cerebral herniation, hypoxic encelphalopathy, aspiration,
thromboembolic events
o Seizures are tonic clonic: 25% prelabor/ 50% labor / 25% after labor (even 7-10
days)
o Tx of seizures: MgSo4 (membrane stabilization), Valium IV
 HELLP: hemolysis, elevated liver enzymes, low platelets
o Usually in the severe pre E classification
o Tx: delivery, MgSo4, hydralazine
 Chronic Htn: <20w EGA, >6w post partum; 1/3 can get superimposed pre E; inc risk of
abruption, DIC acute tubular necrosis, inc. prematurity / IUGR
o TX: procardia (CCB), methyldopa, B blockers, NSTs at 34 weeks

-type 1 diabetes mellitus (T1DM) are at risk for diabetic nephropathy (eg, proteinuria)


and vascular disease (eg, hypertension), which are independent risk factors for
preeclampsia (hypertension of pregnancy with associated proteinuria and/or signs of
end-organ damage).  Therefore, T1DM is one of several conditions that increases the
risk for preeclampsia; others include chronic hypertension, systemic lupus
erythematosus, and multiple gestation (eg, twins).  Patients at high risk for
preeclampsia are prescribed daily low-dose aspirin beginning at 12 weeks gestation.
High-risk patients also require a 24-hour urine collection for total protein at the initial
prenatal visit to establish a baseline (prepregnancy) proteinuria assessment.

DDX: Papilledema& tinnitus, blurry vision


 IIH is most common in obese (BMI ≥30 kg/m ) women of childbearing age, and
2

although pregnancy by itself is not a risk factor, excessive weight gain (as in this patient)
can exacerbate the disorder.
Patients typically have positional headaches that are worse when lying flat, which
increases ICP, and improve with sitting, which decreases ICP.  Pulsatile tinnitus (due
to increased vascular pulsations) and blurry vision (due to increased pressure on the
optic nerves) also commonly occur.  On physical examination, increased ICP can
manifest as optic disc edema (ie, papilledema), abducens nerve (CN VI) palsy (lateral
rectus palsy), and/or visual field deficits.
Diagnosis is with neuroimaging followed by lumbar puncture, which reveals an elevated
opening pressure (>250 mm H2O).  MRI of the brain, often with MR venography to rule
out cerebral vein thrombosis, is the preferred imaging modality and avoids fetal radiation
exposure.  MRI is performed before lumbar puncture to exclude other causes of
elevated ICP (eg, space-occupying mass) that would increase the risk of cerebral
herniation (Choice B).

Heparin-induced thrombocytopenia

Heparin exposure ≥5 days & any of the following:


 Drop in platelet count by >50%
Clinical features  Arterial or venous thrombosis
 Necrotic skin lesions at heparin injection site

 Anaphylactoid reaction after heparin


Diagnosis  Serotonin release assay

 Stop all heparin products


 Start nonheparin medication (

initiation of therapeutic anticoagulation with a


nonheparin anticoagulant (eg, fondaparinux in pregnant patients).
Treatment
-Thrombotic thrombocytopenia purpura can present with
thrombocytopenia and abdominal discomfort; however,
patients typically have fever, purpura, elevated creatinine,
and neurologic findings (eg, weakness, confusion).


 -platelet activation with the release of procoagulant factors, causing venous or
arterial thrombosis.  This patient has symptoms of an acute pulmonary
embolus, including sharp chest pain that worsens with inspiration (ie, pleuritic
chest pain), tachypnea, and decreased breath sounds over the right lung base. 
An embolus in the right lower lobe can also cause radiating pain to the
epigastrium and right upper quadrant.
M
-  Tonometry is used to evaluate for increased intraocular pressure, which can cause
severe eye pain and headache if pressure changes acutely (eg, angle-closure
glaucoma).  In contrast, eye findings associated with IIH (eg, papilledema) are due to
increased intracranial, not intraocular, pressure; therefore, tonometry is not used for
evaluation.

Diabetes in Pregnancy
Priscilla White Classification: not used as much anymore
A1 diet controlled GDM (gestational diabetes mellitus)
A2 GDM controlled with insulin; polyhydramnios, macrosomia, prior stillbirth
B DM onset > 20 yo; duration < 10y
C onset 10-19 yo; duration < 20 y
D juvenile onset dur > 20 y
F nephropathy
R retinopathy
M cardiomyopathy
T renal transplant
 Etiology : impairment in carbohydrate metabolism that manifests during pregnancy ;
50% in subsequent preg ; many get DM later in life.
 Risk Factors: >25 yo, obesity, family history, prev infant >4000 g, prev. stillborn, prev.
polyhydramnios, recurrent Ab
 Assn with: 4x more pre e, 2x more S Abs, inc. infx, inc. hydramnios, c/s, pp hemorrhage,
fetal death
 Fetal anomalies:Transpostion of the great vessels, sacral agenesis, macrosomia, still
birth
 DX: O’Sullivan (50 g glucose) @28 w over 140: fasting <105, 1 hr <190, 2 hr <165, 3 hr
<145
 Management: ADA 1800 – 2200 kcal/d diet; glucose checks, insulin if necessary, deliver
@ 38-40 w oral glucose tolerance test after delivery in six weeks
 Antenatal testing: @ 30-32 w US q 4w (look for IUGR, polyhydramnios), kick counts,
NST, BPP
 Watch for neonatal hypoglycemia

Gestational diabetes mellitus

Pathophysiology  Human placental lactogen secretion

 24-28 weeks gestation


Screening  1-hr 50-g GCT

 3-hr 100-g GTT


 1st line: diet
Management
 2nd line: insulin, glyburide, metformin
 Fasting: ≤95 mg/dL
Target blood glucose goals  1-hr postprandial: ≤140 mg/dL

 2-hr postprandial: ≤120 mg/dL


 Fasting glucose at 24-72 hr
Postpartum management
 2-hr 75-g GTT at 6- to 12-week visit
GCT = glucose challenge test; GTT = glucose tolerance test.
UTI & Pyelonephritis

 Asymptomatic Bacturia: > 100,000 colonies 5% of pregnancies; increased susceptibility


to cystitis and pyelonephritis (15% complicated by bacteremia, sepsis, ARDS); treat as
bacturia because of risks of preterm labor assn with pyelonephritis.
 Causes: Staph saprophyticus, Chlamydia, E Coli, Klebsiella, Pseudomonas,
Enterococcus, Proteus, Coag – staph, group B strep
 Dysuria
 Urinary urgency & frequency
 Hematuria
 Suprapubic/flank pain

 SS UTI: dysuria, frequency, urgency (NEW ONSET ↑ FREQUENT PEEING)
 Dx UTI: U/A + nitrite, WBC esterase, bacteria (contaminated if inc. epithelial cells)
 Tx UTI: (pregnancy): Macrodantin
 SS Pyelonephritis: CVA tenderness, fever, dirty UA (need 2/3 of criteria to diagnose)
 TX Pyelonephritis: IV Ancef until afebrile x 48 hours then 7-14 d po Keflex
 Pyelo is more likely to occur on the R because the uterus is dextrorotated.
Progesterone’s effects cause urinary stasis, which can predispose to pyelonephritis.

Recurrent urinary tract infection

 ≥2 infections in 6 months
Definition
 ≥3 infections in 1 year
 History of cystitis at ≤15 years
 Spermicide use
Risk factors  New sexual partner

 Postmenopausal status
 Urinalysis
Evaluation
 Urine culture
 Behavior modification
Prevention  Postcoital or daily antibiotic prophylaxis

Topical vaginal estrogen for postmenopausal patients


 commonly occur in postmenopausal patients due to estrogen deficiency, which
causes vulvovaginal atrophy, decreased bulk and elasticity of the bladder trigone and
urethra, and increased vaginal pH.  Treatment is with vaginal estrogen.

Infections and Pregnancy

Bacterial Vaginosis: Gardnerella vaginalis


 ss: gray/yellow malodourous discharge – clue cells on wet prep
 tx: Metronidazole (flagyl) in second or third trimester

Group B Strep: Assn with UTI, Chorioamnionitis, endometritis, neonatal sepsis


 2-3/1000 live births assn with GBBS sepsis
 IV pen G or ampicillin in delivery

Herpes Simplex Virus: a DNA virus (HSV 1 and 2)


 If mom has lesions  can give baby viral sepsis on the way out  herpes encephalitis
 Tx: IV Acyclovir, C SECTION if active lesions
Varicella Zoster Virus
 Vertical transmission possible
 If mom gets chicken pox during pregnancy the baby could die
 TX: varicella zoster immune globulin given to mom within 72 hours of exposure; can also
give to infant.

CMV
 SS baby: hepatosplenomegaly, thrombocytopenia, jaundice, cerebral calcifications,
chorioretinitis, interstitial pneumomitis, MR, sensorineural hearing loss, neuromuscular
d/o

Rubella
 SS adults: maculopapular rash, arthralgia, lymphadenopathy for 2-4 d
 SS infant: deafness, CV anomalies, cataracts, MR
 Dx: IgM titers in infant
 Do not give MMR vaccine to pregnant woman
 No tx for rubella

Toxoplasmosis
 First trimester infection: chorioretinitis, microcephaly, jaundice, hepatosplenomegaly
 Adult SS: fever, malaise, lymphadenopathy, rash
 Dx: percutaneous umbilical cord sampling, IgM ab
 Tx: pyrimethamine (<14 w), spiramycin (less teratogenic)

Hepatitis B
 Transm: sex, blood products / transplacental; can cause mild to fulminant hepatitis
 Dx: ab markers: Hbs Ag
 Vaccinated at birth now
Syphilis
 Vertical transmission possible in primary and secondary syphilis
 SS baby: hepatosplenomegaly, hemolysis, LAD, jaundice, saber shins
 Dx: IgM antitreponemal ab
Syphilis manifestations

Primary  Painless genital ulcer (chancre)

 Diffuse rash (palms & soles)


 Lymphadenopathy (epitrochlear)
Secondary  Condyloma latum
 Oral lesions

 Hepatitis
Latent  Asymptomatic

 CNS (tabes dorsalis, dementia)


Tertiary  Cardiovascular (aortic aneurysm/insufficiency)
 Cutaneous (gummas)

HIV
 Vertical transmission possible; AZT decreases chances GREATLY
 Inc transmission with inc viral burden/adv disease

Neisseria gonorrhea
 Transmitted during birth to eye, oropharnyx, ext ear, anorectal mucousa
 Disseminates  arthritis, meningitis
 Screening in early pregnancy
 Tx: ceftriaxone, Suprax po

Chlamydia
 40% babies get conjunctivitis
 10% babies get pneumonitis
 Tx: Zithromax, erythromycin

Septic pelvic thrombophlebitis

 Cesarean delivery
 Pelvic surgery
 Endometritis
Risk factors  Pelvic inflammatory disease
 Pregnancy

 Malignancy
Pathophysiology  Hypercoagulability
 Pelvic venous dilation
 Vascular trauma

 Infection
 Fever unresponsive to antibiotics
 No localizing signs/symptoms
Presentation  Negative infectious evaluation

 Diagnosis of exclusion
 Anticoagulation
Treatment
 Broad-spectrum antibiotics

Hyperemesis Gravidarum

 Morning sickness is found in 80% of women, but usually resolves by 16w


 Hyperemesis: more pernicious vomiting assn with weight loss, electrolyte imbalances,
dehydration, and if prolonged, hepatic and renal damage.
 Tx: maintain nutrition, NS with 5% dextrose, compazine, phenergan, reglan IV/IM; if
needed TPN (total parenteral nutrition)

Coagulation Disorders
 A hypercoaguable state can be due to inc. coag factors (all except 11, 12, dec turnover
time for fibrinogen), endothelial damage, and venous stasis (uterus compresses IVC and
pelvic veins)  increased deep venous thromboses, septic pelvic thromboses and
pulmonary emboli.
 Septic pelvic thrombosis: postpartum, prolonged fever on antibiotics; usually due to
ovarian veins; not likely to lead to emboli; tx is heparin, abx
 Deep Venous Thromboses: SS: edema, erythema, palpate venous cord, tender,
different calf sizes; Dx: Doppler of extremity, venography; Tx: heparin IV (PTT x 2) then
sub Q heparin or lovenox in pregnancy (NO COUMADIN IN PREGNANCY: skeletal
anomalies, nasal hypoplasia); coumadin OK if post partum.
 Pulmonary Embolus: DVT  right atrium  RV  pulmonary arteries  pulm htn,
hypoxia, RHF  death.
SS: sob, pleuritic chest pain, hemoptysis, with signs of DVT
DX: Doppler ext, CXR, ECG, VQ Scan, Spiral CT Pulmonary Angiography
TX: IV heparin then SQ heparin or lovenox (coumadin OK postpartum)

Sickle cell disease in pregnancy

 Baseline 24-hr urine collection/testing for total protein


 Baseline chemistry panel
 Serial urine cultures
Prenatal care  Pneumococcal vaccination
 Folic acid supplement
 Aspirin

 Serial fetal growth ultrasound examinations


 Spontaneous abortion
 Preeclampsia, eclampsia
Obstetric complications  Abruptio placentae

 Antepartum bleeding
 Fetal growth restriction
Fetal complications  Oligohydramnios

 Preterm birth

TERATOGENS

 EtOH: Fetal Alcohol Sd: growth retardation, CNS effects, abnormal facies, cardiac
defects
Tx: alcoholism: aggressive counseling; adequate nutrition
 Caffiene: 80% exposed in first trimester
 Tobacco: Inc. Sab, preterm birth, abruption, dec. birth weight, SIDs, resp disease
 Cocaine: inc. abruption (from vasoconstriction), IUGR, inc PTL; as a child,
developmental delay
 Opiates: (heroin/methadone); the danger is heroin withdrawal, not use  miscarriage,
PTL, IUFD; tx: enroll in methadone program; do not restart methadone if patient has not
used for 48 hours.
Phenytoin: hysical examination shows microcephaly, a wide anterior fontanelle, cleft
palate, and hypoplasia of the distal phalanges. 
n utero exposure to an antiepileptic (eg, phenytoin, carbamazepine, valproate)
eft lip and palate, wide anterior fontanelle, distal phalange hypoplasia, and cardiac
anomalies (eg, pulmonary stenosis, aortic stenosis).  The associated neural tube defects
and microcephaly can also result in developmental delay and poor cognitive outcomes.

ACEi: Use of lisinopril (and other angiotensin-converting enzyme inhibitors) during


pregnancy can cause fetal renal failure and associated oligohydramnios that results in
pulmonary hypoplasia, growth restriction, and limb defects (ie, Potter sequence).

congenital syphilis are asymptomatic at birth; those with symptoms typically have rhinitis
("snuffles"), hepatomegaly, and a maculopapular rash (none of which are seen in this
patient).
Postpartum Care
 Vaginal delivery: pain care/perineal care (ice packs, check for hemorrhage, stool
softener Pelvic rest x 6 w (no douching, tampons, sex); NSAIDS
 C Section: local wound care, narcotics for pain, stool softeners, NSAIDS
 Breast Care: Milk letdown occurs at 24 – 72 hr; if not breast feeding use ice packs, tight
bra, analgesia (breast feeding gives relief)
 Mastitis: oral or skin flora enter a crack in breast skin; can be treated with dicloxacillin;
continue to breast feed.
  Nipple pain that worsens and persists between feedings is commonly due to
nipple injury caused by poor infant positioning and improper latch-on
technique.  On examination, patients can have open, linear areolar abrasions
that cause a bloody-appearing nipple discharge; bruising, cracking, and
blistering may also be present.  Breast engorgement, as seen in this patient
with bilateral, diffusely tender, and engorged breasts, can also develop because
nipple pain limits breastfeeding.

 Contraception: no diaphragms, caps until 6 w; if breast feeding  depo, micronor; not
breastfeeding  OCP, norplant, depo, Orthoevra
 Post Partum Hemorrhage:
o Blood loss vag delivery = 500 cc; c/s = 1000cc (normal – remember, mom’s
plasma volume expands just for this reason!)
o Causes:
 Uterine atony (RF: multip, h/o atony, fibroids) tx: pitocin, methergine, etc.
 Retained products of conception: find on manual exploration of uterus
 Placenta accreta: placenta is stuck in uterine wall
 Cerv/Vag lacs: repair with adequate anesthesia
 Uterine rupture (1/2000) ss: abd pain, “pop” tx: laparotomy and repair if
possible.
 Uterine Inversion (1/2800) RF: fundal placenta, atony, accreta, excess
cord traction tx: manually revert, NTG, Laparotomy
 Post Partum depression:
o Post partum blues: 50%; changes in mood, appetite, sleep, will resolve
o Post Partum depression: 5%; decreased energy, apathy, insomnia, anorexia,
sadness; can get better or proceed to psychosis; tx: antidepressants (SSRIs)
 immediate postpartum period (ie, hours to days) is marked by physiologic changes
that begin immediately after placental delivery:
 Increased oxytocin levels (endogenous and administered) cause uterine
contraction, which compresses placental bed vessels and protects against
postpartum hemorrhage.  As the uterus involutes, it rapidly decreases in size,
becoming firm and palpable 1-2 cm above or below the umbilicus.  Involution
also generates subsequent lochia (shedding of the uterine decidua and blood),
which initially appears bloody with small clots and can continue for several
weeks.
 Increased prolactin levels stimulate breast milk excretion and milk letdown over
the course of hours to days.  Infant suckling further increases maternal prolactin
and oxytocin levels (ie, positive feedback).
 Decreased estrogen and progesterone levels may cause postpartum chills
and shivering, with subsequent mild hyperthermia/low-grade fever in the first 24
hours after delivery.

 Endometritis: a polymicrobial infection invading the uterine wall after delivery;


o SS: fever, inc WBC, uterine tenderness (@ 5-10 d pp), foul discharge
o Look for retained products  do a d & c
o Tx: triple antibiotics until afebrile x 48 hours and pain gone.

Infants of mothers with hepatitis B can safely breastfeed if administration of the hepatitis
B immunoglobulin and initiation of the hepatitis B vaccination series has occurred.  It is
recommended that mothers with hepatitis B and C breastfeed.  However, they should
abstain from breastfeeding if their nipples are cracked or bleeding.

-Galactosemia, caused by impaired infant galactose metabolism, is the only absolute


contraindication to breastfeeding that is neonatal in origi
Breastfeeding contraindications

 Active untreated tuberculosis


 HIV infection*
 Herpetic breast lesions
Maternal  Active varicella infection
 Chemotherapy or radiation therapy

 Active substance use disorder


Infant  Galactosemia

*In developed countries where formula is readily available


POSTPARTUM HEMORRHAGE

Immunologic blood transfusion reactions


Transfusion
Onset* Cause Key features
reaction

 Angioedema, hypotension, respiratory


Within Recipient anti- distress/wheezing,
seconds IgA antibodies shock
Anaphylactic
to directed against  IgA-deficient recipient
minutes donor blood IgA
GIVE EPI

 Fever, flank pain, hemoglobinuria


 Disseminated intravascular coagulation
 Positive Coombs test
 RENAL FAILURE
ABO -Aggressive hydration with IV normal
Within incompatibility
Acute hemolytic saline is indicated for hemodynamic
1 hr (often clerical
error) stabilization and prevention of renal injury. 
In addition to hydration, supportive care
measures (eg, supplemental oxygen,
vasopressors) may be required 

Cytokine
Febrile
Within accumulation
nonhemolytic (most  Fever & chills
1-6 hr during blood
common reaction)
storage
Recipient IgE
Within against blood  Urticaria
Urticarial
2-3 hr product GIVE BENADRYL
component

 Respiratory distress
Donor anti-
Transfusion-related Within  Noncardiogenic pulmonary edema with
leukocyte
acute lung injury 6 hr bilateral pulmonary infiltrates
antibodies
LASIX

 Often asymptomatic
Within Anamnestic  Laboratory evidence of hemolytic
Delayed hemolytic days to antibody anemia
weeks response  Positive Coombs test, positive new
antibody screen
Within Donor T  Rash, fever, gastrointestinal symptoms,
Graft versus host
weeks lymphocytes pancytopenia
*Time after transfusion initiation.

GYNECOLOGY

Mitter
Benign Disorders of Lower Genital Tract

Congenital anomalies:
 Labial fusion: assn with excess androgens  develop abnormal genitalia tx: estrogen
cream
 Imperforate hymen: the junction between the sinovaginal bulbs and the UG sinus is not
perforated  obstructs outflow
o SS: primary amenorrhea at puberty, hematocolpos (blood behind hymen)
o TX: surgery
 Vaginal septums: when vagina forms, the sinovaginal bulbs and mullerian tubercle must
be canalized. If not you get a transverse vaginl septum between lower 2/3 and upper 1/3
 primary amenorrhea
o TX: surgery
 Vaginal agenesis: Rokitansky-Kuster-Hauser Sd: mullerian agenesis/dysgenesis; may
have rudimentary pouch from sinovaginal bulb; Testosterone Insensitivity: 46 xy with no
sensitivity to testosterone (may have undescended testes)
o TX: surgical creation of vagina

TURNER

 Vulvar dystrophy: Hypertrophic: from chronic vulvar irritation = raised white lesions
o TX: cortisone cream bid
o Atrophic: dec estrogen to local tissues (postmenopausal)
o SS: dysuria/parunia, pruritus, Vulvodynia, lichen sclerosis et atrophicus
o Tx : 2% testosterone cream, hydrocortisone cream
 Benign Cysts:
o Epidermal Cyst: occlusion of pilosebaceous duct/hair follicle
 Tx: incision and drainage
o Sebaceous cyst: duct blocked – sebum accumulates
 TX: I & D if infected
o Apocrine Sweat Gland Cyst: on mons or labia  occludes glands 
superinfection  hidradentitis suppurative  I & D, Doxycycline
o Bartholin’s gland Cyst: 4 or 8 o’clock on labia majora

 TX: sitz baths, infx – I & D / word catheter

This patient has hidradenitis suppurativa, a chronic inflammatory condition that


involves recurrent occlusion of hair follicles in intertriginous regions (eg, groin, axilla). 
Hidradenitis suppurativa commonly develops in women age 20-40; risk factors include
obesity, tobacco use, and family history.

 Cervical Lesions
o Congenital anomalies: DES exposure in utero = 25% congenital anomalies, clear
cell adenocarcinoma 1%
o Cervical Cysts: dilated retention cysts: nabothian cysts = blockage of
endocervical gland @ 1 cm – asx, no TX
o Mesonephric Cysts: (remnants of wolfian/mesonephric ducts) deeper in stroma
o Polyps: broad based = can have intermittent/post coital bleeding; usually
removed cervical fibroids = intermenst bleeding, dysparunia, bladder/rectal
pressure/ r/o cerv can
o Cervical Stenosis: congenital or after scarring (surgery/radiation) or secondary to
neoplasm or polyp; if asymptomatic, leave alone; if causes menstrual problems,
remove; gently dilate scarring.

Mittersmitz pain: during ovulation


.  Abdominal examination shows mild left lower quadrant tenderness with no rebound. 
Pelvic examination shows a small, anteverted uterus with tenderness over a slightly
enlarged left adnexa; the right adnexa is normal.  Urine pregnancy test is
negative.  Pelvic ultrasound shows the left ovary with a 3-cm cyst with several
subcentimeter cysts in the periphery.  There is normal Doppler blood flow with a small
amount of free fluid in the posterior cul-de-sac

DDX
 An endometrioma arises from ectopic endometrium within the ovary that bleeds and
forms a hematoma, which appears on ultrasound as a homogenous ovarian cyst with a
ground-glass appearance.
-Ovarian torsion results from a large ovarian mass twisting around the infundibulopelvic
ligament causing occlusion of the ovarian vessels, resulting in ovarian ischemia.  It often
presents with severe unilateral pain with associated nausea and vomiting; however,
ultrasound typically shows absent blood flow to the adnexa,
- tuboovarian abscess presents with fever, diffuse lower abdominal pain, and a complex,
multicystic adnexal mass with thickened walls on ultrasound.

Fibroids

 Fibroids = Estrogen dependant local proliferation of smooth muscle cells, usually occur in
women of child bearing age and regress at menopause; African American are at higher
risk; has a pseudocapsule of compressed muscle cells; are found in 20-30% American
women at age 30
 SS: menorrhagia (submucous), metrorrhagia (subserous, intramural), pressure sx (from
pressing against bladder), infertility; 50% are asymptomatic.
 Parasitic fibroids: get their blood supply from the omentum.
 Histologic Changes:
o Hyaline Change
o Cystic Change
o Calcific change
o Fatty Change
o Red/white infarcts
o Sarcomatous change (most rare)
 In pregnancy are at increased risk for Sab, IUGR, PTL, Dystocia; may grow during
pregnancy
 Med TX: Depo provera, Lupron (GnRH agonist), Danazol Transexemic ACID!!! (OK IN
PREG)
 Surg Tx: momectomy(only for fertility purposes), hysterectomy indicated when anemic
from bleeding, severe pain, size > 12 w, urinary frequency, growth after
menopause, new role for embolization by interventional radiology

Warm compresses and massage are indicated in the management of a clogged


lactiferous duct, which can present with unilateral breast pain; however, the mass is
typically located in the subareolar region and has no associated axillary
lymphadenopathy.

Endometrial Hyperplasia
 Endometrial hyperplasia: abnormal proliferation of gland/stromal elements;
overabundance of histologically normal epithelium
o Simple without atypia: 1% cancer- Provera
o Complex without atypia: 3% cancer- Provera
o Simple with atypia: 9% cancer- Provera vs. TAH
o Complex with atypia: 27% cancer- TAH
o RF: unopposed estrogen, PCO, granulosa/theca tumors
o DX: endometrial biopsy

Endometrial biopsy indications

 Abnormal uterine bleeding


Age >45
 Postmenopausal bleeding
Abnormal uterine bleeding PLUS:
 Unopposed estrogen (obesity, anovulation)
Age <45
 Failed medical management

 Lynch syndrome (hereditary nonpolyposis colorectal cancer)


Atypical glandular cells on Pap test NEED AGC on Pap testing is investigated
Age >35 with colposcopy, endocervical curettage, and endometrial biopsy 
to evaluate the ectocervix, endocervix, and endometrium

Endometrium: 5 OR MORE, CUT THAT _____


 Pap test for cervical cancer, regardless of when the last Pap test was
performed, because cervical cancer can present with light vaginal bleeding and
has a peak incidence in women age ~50.
 endometrial biopsy or transvaginal ultrasound for endometrial cancer because
the most common presentation of endometrial cancer is PMB and the risk for this
cancer increases with age and BMI.

-------

Endometrial hyperplasia, a precursor to endometrial cancer, occurs due to unregulated


endometrium proliferation.  Obesity is a major risk factor for endometrial hyperplasia
because adipose tissue increases peripheral conversion of androgens to estrone,
thereby increasing estrogen levels and causing unopposed uterine estrogen exposure

Obese women have increased estrogen levels because of the increased peripheral


conversion and aromatization of androgens to estrone in adipose tissue.  The
increased unopposed estrogen levels cause and compound chronic anovulation,
which results in continued stimulation of the endometrium.  The endometrial lining
continues to proliferate and becomes increasingly disordered, resulting in hyperplasia.
(Choice A)  Chronic endometritis, a polymicrobial infection within the endometrial cavity,
can cause intermenstrual bleeding due to sloughing of infected tissue.  It does not affect
ovulation; therefore, it does not increase the risk of endometrial hyperplasia.
(Choice B)  Progesterone, provided by either endogenous (eg, ovulation) or exogenous
(eg, progestin-releasing intrauterine device) sources, decreases endometrial proliferation
and promotes endometrial differentiation.  Therefore, it is protective against endometrial
hyperplasia/cancer and can be used in treatment of select cases of endometrial
hyperplasia.
(Choices C and E)  Uterine leiomyomata (ie, proliferative myocytes that distort the
uterine cavity) and adenomyosis (ie, myometrial invasion of endometrial glands) typically
cause heavy, regular bleeding because they increase the surface of the endometrial
cavity.  They do not increase the thickness of the endometrial lining; therefore, neither
increases the risk of endometrial hyperplasia.
(Choice D)  Endometriosis (ie, ectopic implantation of endometrial glands) typically
causes heavy, painful menses (rather than abnormal bleeding) due to inflamed
endometrial tissue on the peritoneum.  Women with this condition have a slightly
increased risk of epithelial ovarian cancer; however, endometriosis does not increase
the risk of endometrial hyperplasia.

Endometriosis INFERTILITY

intensifies a few days before the patient's menstruation and improves toward the end of
her cycle. Examination shows tenderness in the posterior vaginal fornix, decreased
uterine mobility, and thickening of the uterosacral ligaments.  No adnexal masses are
palpate. Normal anatomy
 Adenomyosis: Endometrium in myometrium
o Ususally a 30 yo multiparous woman with heavy painful periods, enlg tender
uterus described either as boggy/soft or woody/firm and pelvic heaviness
o Rx: hysterectomy / analgesics
o The tissue does not undergo proliferation phase of cell cycle.
 Pelvic Endometriosis: presence of endometrial glands outside of endometrium
o Theories
 Sampson’s reflux menstruation: most likely
 Coelomic metaplasia: irritant to peritoneum
 Family history / genetic
 Immunologic
 Lymphatic and vascular mets
 Iatrogenic dissemination (ie:you see it on the other side of a c section
scar)
o Induces fibrosis which causes pelvic pain
 with a cervix that appears laterally displaced;
o
Pathogenesis  Ectopic implantation of endometrial glands

 Dyspareunia
 Dysmenorrhea
 Chronic pelvic pain
Clinical features  Infertility
 Dyschezia

 Cyclic dysuria, hematuria


 Immobile uterus
 Cervical motion tenderness
Physical examination  Adnexal mass

 Rectovaginal septum, posterior cul-de-sac, uterosacral ligament nod


Diagnosis  Direct visualization & surgical biopsy

Treatment  Medical (oral contraceptives, NSAIDs) 


 Surgical resection

o SS: pain, infertility, bleeding/ovarian dysfunction, hematochezia/ hematuria,
dyspareunia (pain with sex)
o Can be on peritoneum, ovary (chocolate cysts), round ligament, tube, sigmoid
colon

o
o DX: laparoscopy
o TX:
 NSAID
 OCP/Provera
 Lupron (GNRH agonist) – pseudomenopause
 Laser surgery/coagulation of implants, TAH/BSO
Ovarian Cysts

 Usually follicular from failure of follicle rupture; disappear in 60 d


 3 – 8 cm
 Types:
o Corpus luteum cysts (firm/solid)
o Cystic/hemorrhagic (hemoperitoneum)
o Theca lutein (bilateral, filled with straw fluid; high bHCG)
 DX: ultrasound, CA125 in cases suspect for epithelial ovarian cancer
 DiffDX: ectopic, tuboovarian abscess, torsion, endometriosis, neoplasm
 TX: if premenopausal, can observe if under 8cm; If postmenopausal (any size) or
premenopausal need laparoscopy vs. laparotomy for cystectomy or oopherectomy

Treatment of STDs

 Chlamydia trachomatis:
o DX – Direct fluorescent Ab
o Tx: doxycycline 100 mg bid x 7 d or Azithromycin 1 g po (one dose)
 N. Gonorrhea:
o DX: gram stain, culture
o RF: low SES, urban, nonwhite, early sex, prev gon infx
o Treat both partners
o TX: Rocephin 250 mg IM or Cipro 500 mg po or Floxin 400 mg po
o Usually transfers male to female more than female to male.
 Syphilis: Treponema pallidum
o DX: dark field microscopy
o TX: (<1 yr duration) Pen G 2.4 million U IM (>1yr duration) 2.4 mill U IM x 3
doses (see ob section for full description)
 Herpes Simplex Virus: first episode – Acyclovir/Famciclovir/Valcyclovir; 66% HSV-2
33% HSV-1 of genital herpes; vesicles rupture in 10-22 d leaving a painful ulcer; can use
antivirals also as suppressing agents as the virus hangs out in the dorsal root ganglion.
 HPV:
o Types 6/11 = genital warts
o Types: 16,18,31 = cervical cancer
o TX: podofilox, cyrotherapy, podophyllin rein, TCA, Aldara cream
 Chancroid: casued by Haemophilus ducreyi; is a painful soft ulcer with inguinal
lymphadenopathy; tx with Ceftriaxone 250 IM x once or Azithromycin 1 g once po or
Erythromycin; treat partner.
 Lymphogranulona venerum: primary = papules/shallow ulcer; secondary = painful
inflammation of inguinal nodes with fever, h/a, malaise, anorexia; Tertiary = rectal
stricture/rectovaginal fistula/ elephantiasis TX: doxycycline 100 mg po bid x 21 d
 Molluscum contagiosum: pox virus from close contact; 1-5 mm umbilicated lesion
anywhere but the palms or soles; are asymptomatic and resolve on their own
 Phthris pubis/sarcoptes scabei: Lice and scabies, respectively; TX: lindane/Kwell

Infectious genital ulcers


Painful Herpes simplex  Pustules, vesicles, or small ulcers on erythematous base
virus  Tender lymphadenopathy
 Systemic symptoms common
several tender, ulcerated lesions with circular borders on the
inside of the left labia minora.

Haemophilus  Larger, deep ulcers with gray/yellow exudate


ducreyi  Well-demarcated borders & soft, friable base
(chancroid)  Severe lymphadenopathy that may suppurate
Treponema
 Usually single ulcer (chancre)
pallidum
 Indurated borders & hard, nonpurulent base
(syphilis)

Painless Chlamydia
trachomatis
 Initial small, shallow ulcers (often missed)
serovars L1-L3
 Then painful & fluctuant adenitis (buboes)
(lymphogranuloma
venereum)
This patient's multiple, painful genital ulcers are consistent with a genital herpes simplex
virus (HSV) infection.  Patients with a primary infection often have systemic symptoms (eg, fever)
and develop a tender inguinal lymphadenopathy.  HSV evolves from vesicles to open ulcers;
patients with ulcers often have associated dysuria and sterile pyuria (eg, white blood cells
[WBCs] but no bacteria on urinalysis) due to urethral and vulvar inflammation and passage of
urine over the open lesion.  In addition, some patients may develop acute urinary retention (eg,
suprapubic fullness) due to either reluctance to urinate or from a lumbosacral neuropathy that can
complicate the infection.
The appearance of genital HSV lesions can vary and mimic other disease processes as the
lesions change from vesicles to ulcers.  Therefore, a suspected clinical diagnosis of genital HSV
requires laboratory confirmation via viral culture or PCR testing.  Viral culture is most effective in
patients with active HSV lesions (such as this patient) but has decreasing sensitivity as lesions
heal.
(Choice A)  Haemophilus ducreyi is a sexually transmitted infection that causes chancroid, which
can cause multiple painful ulcers and tender inguinal lymphadenopathy (less common in
women).  However, the ulcers have a gray/yellow exudate and a friable base, and the lymph
nodes classically undergo supuration (eg, pus).   Diagnosis is via bacterial culture; Gram stain
typically show gram-negative rods.
(Choice B)  KOH wet mount microscopy is used to diagnose vulvovaginal candidiasis, which can
present with a pruritic, erythematous vulvar rash and dysuria; however, there are no associated
genital ulcers.
(Choice C)  Chlamydia trachomatis can cause dysuria and sterile pyuria (ie, WBCs but no
bacteria) due to urethritis, but patients typically have concomitant acute cervicitis (eg, cervical
friability, mucopurulent discharge).  Lymphogranuloma venereum is caused by C
trachomatis serovars L1-L3 and presents with small, painless ulcers followed by painful,
suppurative inguinal lymphadenopathy (buboes).
(Choice D)  Nontreponemal serologic testing (ie, rapid plasma reagin) is used for evaluation of
syphilis; primary syphilis typically presents with a single painless ulcer (ie, chancre) and bilateral,
nontender lymphadenopathy.

Vaginitis

 Candida:
o RF: Abx, DM, Pregnancy, immunocompromised
o SS: burning, itching, vulvitis, cottage cheese discharge, dysparunia
o DX: wet prep KOH = branching hyphae
o Exam: white plaques with or without satellite lesions
o TX: over the counter creams work well (monistat); if resistant, Diflucan 150 mg
po x once


o CHECK A1C on PATIENT!
o
 Trichomonas: unicellular flagellated protozoan
o SS: itching, inc. discharge (yellow/gray/green), frothy
o Exam: strawberry cervix, foamy discharge
o DX: see the buggers zipping all over your wet prep
o TX: Flagyl 500 mg po bid x 7 d/ partner condom x 2 w
o Note: avoid flagyl in frist trimester
 Bacterial vaginosis: Gardnerella vaginalis
o SS: odorous discharge
o DX: whiff test by adding KOH; see clue cells on wet prep (spotty squamous cells)
o TX: flagyl 500mg bid x 7 d
o Not an STD

 Atrophy
o SS: burning d/c on sex
o RF: post menopausal
o TX: estrogen

PID
Girl with FEVER and large, thick-walled, multiloculated mass filled with debris obliterating
the right adnexa. Leukocytosis. Neg pregnancy

Organisms: Neisseria, Chylamadia, Mycoplasmia, Ureaplasma, Bacterioides, among others


SX: diffuse lower abdominal pain, vaginal discharge, bleeding, dysuria, dyspareunia, CMT,
adnexal tenderness, GI discomfort
DX: Cervical Motion Tenderness, Adenexal tenderness, discharge, fever, elevated WBC,
ESR
Lab: cultures, pelvic U/S if mass palpated, rise in WBC count
TX: Ceftiaxone 2 g IV q 12, Doxycycline 100 mg IV or Clindamycin – Gentamycin
Usually tx for 48 hrs IV then if afebrile change to Doxycyclin 100 mg po bid x 14 d

TOA: Tubo Ovarian Abcess: persistent PID progresses to TOA in 3-16% of the time
Adnexal mass/fullness (not walled off like true absess)
DX: U/S, Pelvic CT if obese, increase WBC with a shift to the left, increase ESR
TX: Hospitalize for IV antibiotics (Triples: ampicillin, gentamycin, clindamycin) if TOA ruptures or
doesn’t resolve with antibiotics then surgery.
Pelvic inflammatory disease

Symptoms  Abnormal bleeding and abd PAIN

 Multiple sexual partners


 Age 15-25
Risk  Previous pelvic inflammatory disease
factors  Inconsistent barrier contraception use

 Partner with sexually transmitted infection


 Fever >38.3 C (>100.9 F)
Physical  Cervical motion, uterine, or adnexal tenderness
examination
 Mucopurulent cervical discharge
 Inpatient: IV broad-spectrum antibiotics
Treatment
 Outpatient: PO broad-spectrum antibiotics
 Tuboovarian abscess
 Infertility
Complications  Ectopic pregnancy

 Perihepatitis
Ddx:  Ruptured ovarian cysts typically cause sudden-onset, severe, lower abdominal
pain provoked by activity.  However, ruptured ovarian cysts typically occur in the middle
of the menstrual cycle (this patient is menstruating) and do not cause a high fever.

ENDOMETRITIS: usually after some type of instrument disruption of the uterus: C-section,
vaginal delivery, D & E/C, IUD)
DX: endometrial or endocervical culture will result in skin, GI, repro flora
TX: Doxycycline vs. IV abx

TOXIC SHOCK SYNDROME: vaginal infection that is not associated with menstruation
Can be assoc with delivery, c-sections, post partum Endometritis, sab or laser tx of coac
Staph aureus produces epidermal TSS T-1 that produces fever, erythema rash
desquamation of palmer surfaces and hypotension. Also see GI disturbances, myalase;
mucus membrane hyperemia, change in mental status
Labs: increased BUN/CR, decreases plt; but neg blood cultures
TX: always hospitalize… may need ICU and give IV fluids and / or pressors. ABX do not shorten
the length of the acute illness but does decrease the risk or recurrence.

BLADDER ANATOMY
- Detrusser and urethra = smooth muscle
- Internal spincter is at urethrovesical jxn
- Incontinence = intraurethral < intravesical pressure
- PSNS (S2,3,4) allows micturition : CHOLINERGIC RECEPTORS
- SNS – hypogastric n. T 10 – L2 prevents urination by contracting bladder neck and internal
spincter : NE RECEPTORS
- Somatic controls external spincter (pudendal nerve)

PELVIC RELAXATION: damage to the anterior vaginal wall leading to cystocele, endopelvic
fascia leading to rectocele or enterocele or stretching of cardinal ligaments which can lead to
uterine prolapse
DX: mostly PE : called a POP Q, which is a graph on which certain points corresponding to
lengths of the vagina and where it moves on valsalva are graphed. This tells you where the defect
is, so you know the appropriate therapy from it.
SX: pain, pressure, dyspareunia, incontinence, bowel or bladder dysfunction
Causes: anything that will cause chronically increased abdominal pressure: cough, straining,
ascites, pelvic tumors, heavy lifting
RF: aging, menopause, traumatic delivery, associated with multiparity
PE: pelvic exam shows the amount of descent of the structure into the vagina and thus
determines the degree of relaxation: (POP Q)
Stage 1 – upper 2/3 of vagina
Stage 2 – to the level of the introitus
Stage 3 – outside of the vagina
TX: kegels (contraction of levator ani muscle, instructed by physician), estrogen replacement,
vaginal pessaries, surgery

INCONTINENCE:

URGE INCONTINENCE: aka detrussor instability


SX: urgency, often can not make it to the bathroom
Causes: foreign body, UTI, stones, CA, diverticulitis
Dx: based on history, can be shown on urodynamics (which is a catheter in the bladder, rectum
and a machine to measure the difference. The bladder is filled with normal saline and response to
that is measured.)
Urodynamics shows: involuntary/uninhibited bladder contractions
TX: Kegle exercises, anticholerginics (ditropan, amytriptaline), muscle relaxants, beta agonists,
estrogen replacement- surgery is not used here, more medical therapy is appropriate

STRESS INCONTINENCE: URETHRAL HYPERMOBILITY. Stress urinary incontinence


occurs due to weakened pelvic floor muscles that cause urethral hypermobility and
reduced bladder support.
SX: involuntary loss of urine when there is an increased abdominal pressure mostly from
sneezing, coughing, laughing which transmits pressure to the urethra
Mech: Intrinsic spincter defect, hypemobile bladder neck, pelvic relaxation--
Causes: trauma, neurologic dysfunction, associated with multiparity
TX: Keglel exercises, alpha agonists, estrogen cream, retropubic urethropexy (which is a
surgery where the periurethral tissue is joined with the Cooper’s ligament – called a Burch) or
Trans Vaginal Tape procedure (the periurethral tissues are raised towards the abdominal wall
using a mesh sling- placed under local anesthesia)

-tress urinary incontinence, a common cause of incontinence in women.  The bladder


and urethra are normally maintained in the appropriate anatomic position by the pelvic
floor (levator ani) muscles.  However, with chronic pressure or injury to the pelvic floor
muscles—commonly due to childbirth (ie, multiparity), obesity, or chronic high-impact
exercise such as jogging (as in this patient)—women can develop pelvic floor muscle
weakness.
Substantial weakness of the pelvic floor muscles can result in urethral hypermobility,
in which the urethra abnormally moves with increased intraabdominal pressure (eg,
jogging, coughing) and is unable to fully close.  In addition, inadequate bladder support
can develop (ie, prolapse), as evidenced by the anterior vaginal bulge (cystocele) in this
patient.  Due to both the urethra's inability to fully close and increased bladder pressure,
patients have intermittent leakage of urine when intraabdominal pressure is increased
(ie, Valsalva).
Urinalysis and postvoid residual volume are normal (<150 mL in women, <50 mL in
men).  First-line treatment includes pelvic floor muscle exercises (eg, Kegel exercises)
and lifestyle modifications (eg, weight loss).  Unresponsive cases may require use of a
pessary or midurethral sling surgery.
-------

OVERFLOW INCONTINENCE:
SX: dribbling, urgency, stress
Mech: underactive detrussor leading to poor or absent bladder contractions
Cause: DM, drugs, fecal impaction, MS, neurologic
TX : treat underlying cause, Hytrin, bethanechol, intermittient cath, dantroleen
DX: urodynamics, post void residual (after you pee, you place a catheter to see how much urine
is left in the bladder- over 100 cc is abnormal)

URINARY FISTULA: produces continuous urine leakage commonly seen following pelvic
surgery/radation
RF: PID, radiation, endometriosis, prior surgery
DX: Methylene blue dye injection into the bladder—place a tampon in the vagina- if it’s a
vesicovaginal fistula the tampon will be blue, indigo carmine dye given IV with a tampon in vagina
—if it’s a ureterovaginal fistula the tampon will be blue
TX: surgery but must wait 3 – 6 months to repair postsurgical fistulas

Urethral diverticulum
Definition
 Urethral mucosa herniated
into surrounding tissue

Clinical features
 Dysuria
 Postvoid dribbling
 Dyspareunia
 Anterior vaginal wall mass

Diagnostic testing
 Urinalysis
 Urine culture
 MRI of the pelvis
 Transvaginal ultrasound

U. Diverticulium A 2-cm tender anterior vaginal mass is present and palpation of the mass
expresses a bloody discharge from the urethral meatus. ..  there is a 2-cm mass on the
anterior vaginal wall.  The mass is tender to palpation and expresses a purulent discharge
from the urethra

 urethral diverticulum, an abnormal area of localized urethral mucosa due to


recurrent periurethral gland infections along the anterior vaginal wall.  The recurrent
infection and inflammation of the urethral tissue creates the abnormal outpouching that
can collect and store urine, resulting in postvoid dribbling and recurrent lower urinary
tract infections (eg, dysuria).  Infection of the diverticulum can also lead to pain, often
presenting as dyspareunia or a tender anterior vaginal wall mass with an associated
expressed purulent or bloody urethral discharge.  Diagnosis is confirmed with a pelvic
MRI, and treatment is via surgical excision of the diverticulum.
(Choice A)  Vesicovaginal fistula, an abnormal tract between the bladder and vagina,
typically presents with constant, rather than intermittent, leakage of urine due to
continuous drainage of the bladder into the vagina.  In addition, there is no associated
anterior vaginal wall mass or purulent urethral discharge.
(Choice B)  Acute infection of the lower urinary tract (ie, cystitis, urethritis) can cause
dysuria and urinary incontinence; however, patients often also have urgency and
frequency.  There is no association with a tender anterior vaginal wall mass.
(Choice C)  Stress urinary incontinence, caused by hypermobility of the urethra and
bladder neck, is common in multiparous and obese patients, such as this one.  Patients
typically have urinary leakage with increased intraabdominal pressure (eg, Valsalva),
which is not seen in this case.
(Choice D)  Overflow incontinence, which is due to impaired bladder detrusor
contractility, can occur in postmenopausal women.  Patients typically have constant
dribbling of urine from incomplete bladder emptying and chronic urinary retention (ie,
overflow).  This diagnosis is unlikely because this patient has a normal postvoid residual
(ie, no retention).

Urethrocele

-
ENDOCRINOLOGY

PUBERTY: secondary sex characteristics, growth spurt, achievement of fertility


1. Adrenarche (6-8 yo): regenerates zona reticularis that produces DHEA-S, DHEA,
androsteinone
2. Gonadarche (yo): pulsatile GnRH secretion goes to ant pituitary to secrete LH, FSH
3. Thelarche (breast, 11 yo): Tanners stages
4. Pubarche (12 yo): pubic hair, Axillary hair
5. Growth spurt: (9-13 yo): increase GH and somatomedian – C result in peak height velocity,
increase estrogen levels, fusion of growth plate
6. Menarche: (12 – 13 yo): anovulatory period up to 1 year, may take 2 years to have regular
cycle, delayed in athletes

Two pneumonics: (pick your favorite) “breast hair grow bleed” or “boobs pubes pits and pads”

TANNER STAGES
Breast Hair
1. Prepubertal 1. prepubertal
2. Breast bud 2. presexual hair
3. Breast elevation 3. Sexual hair
4. Areolar Mound 4. Mid-escutcheon
5. Adult Contour 5. Female escutcheon

MENOPAUSE: cessation of menstruation


Onset – usually 50- 51 years
- if <40 yrs premature menopause
- if <35 premature ovarian failure (idiopathic, send genetic studies)
SX: irregular menses, hot flashes secondary to decreased estrogen, mood changes, depression,
lower urinary tract atrophy, genital changes, osteoporosis
LABS: FSH > 40, elevated LH, decreased estrogen resulting in decreased negative feedback
DX: H & P (PE shows decreased breast size with vaginal, urethral, and cervical atrophy 2 to
decreased estrogen)
TX: Hormone replacement (HRT) primarily estrogen and progesterone if pt has uterus; calcium,
Vit D, exercise to counter the decreased osteoclast activity: Estrogen cream to counter act
vaginal atrophy.
Contraindications: Vaginal bleeding, thromboembolic dz, breast ca, uterine ca
Unopposed estrogen (estrogen without progesterone in women without a uterus) results
in endometrial hyperplasia and CA.
Consequenses of decreased estrogen:
- unfavorable lipid profile that could result in stroke and MI
- Increased bone resorption b/c estrogen decreases osteoclast activity predisposing to hip fract.
- Atrophy of skin and muscle tone.

WHI Study: What are all these questions about estrogen and progesterone on the news? In
women with active heart disease, estrogen and progesterone (prempro) increases the remote risk
of stroke and DVT. There were problems with this study, however.

There are no problems taking estrogen alone when you don’t have a uterus.

PRIMARY AMENORRHEA: Estrogen  gives breasts; Y chromosome makes Mullerian


Inhibitory Factor- no uterus if MIF present.
1. No Breasts, + uterus: no estrogen
a. FSH high: ovarian failure (hypergonadotropic hypogonadism)
i. Turner’s : ovaries undergo rapid atresia
ii. Mosaic
iii. 17 hydroxylase def : MIF produced so no female internal organs
iv. Pure Gonadal dysgenesis
b. FSH low: insufficient GnRH, hypo pituitarism, Swyer’s Sd: Gonadal agenesis,
46xy, testes do not develop b/c MIF not released, infertility, external female
genitalia, no breast.
2. +Breast, – uterus: estrogen + MIF
a. Rokitansky Kuster Hauser: uterovaginal agenesis with other anomalies 46xx
b. Androgen insensitivity: 46xy, testicular feminization, no receptors for
testosterone, MIF secreted therefore no mullerian structures.
Androgen insensitivity syndrome

Pathophysiology  X-linked mutation in androgen receptor

 Genotypically male (46,XY karyotype)


 Phenotypically female
 Breast development
Clinical features  Absent or minimal axillary & pubic hair
 Female external genitalia
 Absent uterus, cervix, & upper one-third of vagina

 Cryptorchid testes

Management  Gender identity/assignment counseling


 Gonadectomy (malignancy prevention)
i.
3. –Breast, – uterus: xy (no steroids) but phenotypically female, no internal female organs.
a. 17 hydroxylase def (steroid synthesis) in XY
4. +Breast, – uterus:
a. Imperforate hymen – solid membrane across introitus, pelvic/abd pain from
accumulation of menstrual fluid – hemato colpos.
b. Trans vaginal septum – failure to fuse mullerian determined upper vagina and
UG sinus found at mid vagina tx: surgery
c. Vaginal agenesis RKH, mullerian agenesis/dysgenesis uterial of partial
vaginagenesis, no patent vagina, 46xx, and ovaries and uterus on U/S. Tx:
surgery.
Inperforate HYMEN
The initial menstrual cycles in adolescents are irregular and anovulatory. 
The abnormal uterine bleeding (AUB) that follows menarche is due to immaturity of
the developing hypothalamic-pituitary-gonadal axis that produces inadequate
quantities and proportions of gonadotropin-releasing hormone, and subsequently FSH
and LH, required to induce ovulation.  In the absence of ovulation, menstrual cycles lack
their regular periodicity.  The endometrium builds up under the influence of estrogen, but
without the influence of progesterone, the cue to slough the endometrium is lacking and
menstrual-like bleeding occurs due to estrogen breakthrough bleeding.  Normally,
progesterone is produced in increased amounts by the corpus luteum following
ovulation, and withdrawal of this progesterone as the corpus luteum degenerates results
in menses.
In patients with AUB, bleeding after exogenous progesterone administration confirms
normal endogenous estrogen production and proliferative endometrium.  It also
eliminates causes of estrogen deficiency (eg, primary ovarian insufficiency), endometrial
abnormalities (eg, intrauterine adhesions),  and outlet tract abnormalities (eg,
imperforate hymen).  Treatment of AUB in adolescents with either progestin-only or
combination estrogen/progestin oral contraceptives is indicated when bleeding is
disruptive to activities, is heavy, or results in anemia.  Anovulation typically resolves 1-4
years postmenarche, after which menstrual cycles normalize.
(Choices A and C)  Excess LH secretion and androgen excess are characteristic of
polycystic ovary syndrome.  Clinical features include irregular menses, signs of
hirsutism, and multiple small "pearl-string" ovarian cysts on ultrasound.  This patient has
no signs of hirsutism and shows normal ovaries on ultrasound, making this diagnosis
less likely.
(Choice B)  Estrogen deficiency, characteristic of primary ovarian insufficiency, typically
presents with secondary amenorrhea.  Patients with estrogen deficiency do not have
bleeding after progesterone stimulation due to the lack of proliferative endometrium.
(Choice E)  Intrauterine adhesions (eg, Asherman syndrome) are a cause of secondary
amenorrhea typically due to previous uterine instrumentation (eg, dilation and curettage)
or infection (eg, endometritis).  In these patients, a progesterone withdrawal bleed does
not occur due to the lack of endometrial tissue.
SHEEHAN VS ASHERMAN
SHEEHAN: LOW LH nad FSH!!!!!
ASHERMAN: NORMAL

Intrauterine adhesions

 Infection (eg, septic abortion, endometritis)


Risk factors
 Intrauterine surgery (eg, curettage, myomectomy)
 Abnormal uterine bleeding
 Amenorrhea
Clinical features  Infertility
 Cyclic pelvic pain

 Recurrent pregnancy loss


Evaluation  Hysteroscopy

Educational objective:
The initial menstrual cycles in adolescents are irregular and anovulatory due to
hypothalamic-pituitary-gonadal axis immaturity and insufficient secretion of
gonadotropin-releasing hormone.

SECONDARY AMENORRHEA:
 Must do a good H&P to check for stresses, wt loss/gain, drugs, exercise, upt, Estradiol
level, progesterone challenge
 Enough estrogen (bleeds with progesterone challenge) check FSH, LH, PRL
o LH high think PCO
o LH wni think hypothalamic amenorrhea so stress, exercise, post pill
o PRL increased think prolactinoma, hypothyroidism, prenothrazines, pregnancy
 No estrogen (no bleed with progesterone challenge) check FSH, LH, PRL
o FSH high think ovarian failure, resistant ovarian syndrome
o FSH low – wnl check MRI/CT for pituitary tumors, Sheehan’s Simmans syndrome
o Could also be post surgery problems:
 Asherman’s following D&C
 Cervical stenosis following CKC

This patient has a history of cancer, amenorrhea, and signs of estrogen


deficiency (eg, vaginal dryness), consistent with ovarian failure secondary
to chemotherapy.  Hypergonadotropic hypogonadism in women age <40, also
known as primary ovarian insufficiency, occurs due to the absence of developing
follicles

-Asherman syndrome is a structural cause of secondary amenorrhea due to intrauterine


adhesions from endometrial infection (eg, endometritis) or instrumentation (eg, dilation
and curettage).  Hormone levels are normal.

- Hypothyroidism can present with menstrual dysfunction, fatigue, and hair changes,
without signs of hypoestrogenism.  Findings include an enlarged thyroid, elevated TSH,
and low T3/T4 levels.  In hypothyroidism, low T3/T4 increases thyrotropin-releasing
hormone (TRH) secretion from the hypothalamus.  Increased TRH stimulates prolactin
secretion, which has inhibitory effects on GnRH production, thereby decreasing FSH and
LH levels.
 ↓Thyroid: Hypothyroidism causes decreased responsiveness of the pituitary to
GnRH, leading to decreased gonadotropin (ie, FSH, LH) secretion.
 Thyrotropin-releasing hormone, which is upregulated in hypothyroidism,
stimulates the pituitary secretion of prolactin.
 Hyperprolactinemia, whether due to hypothyroidism or as an isolated disorder,
further suppresses gonadotropin secretion.

Swyer’s Syndrome: 46xy, gonadoagenesis, w/o testes no MIF yielding female genitalia but no
estrogen so no breasts.
Kallman’s Syndrome: absence of GnRH and anosomia. Pts have breast and uterus
Testicular Feminization: 46xy insensitive to testosterone. MIF so no internal female genital
structures + estrogen so has breasts.

PMS
2nd ½ of cycle
Probable Causes: abnormal estrogen/progesterone balance, increase PG production, decrease
endogenous endorphins; disturbance in renin-angiotensin-aldosterone system
DX: 5 of 12 symptoms (including 1 of the first four)
SX:
1. Decreased mood
2. Anxiety
3. Affective Liability
4. Decrease interest
5. Irritability
6. Concentration difficulty
7. Decreased energy
8. Change in appetite
9. Overwhelmed
10. Edema
11. Edema
12. Weight gain
13. Breast Tenderness
TX: avoid caffeine, etoh, tobacco, low sodium diet, weight reduction, stress management.
Drugs: NSAIDS, OCPs, lasix, calcium, vit E, SSRI

DYSMENORRHEA: pain and cramping during menstruation that interferes with the acts of daily
living.
Primary – presents <20 years b/c of increased PG occurs with Ovulatory cycles
Secondary – Endometriosis, Adenomyosis, fibroids, cervical stenosis (congenital, trauma,
surgery, infection), adhesions (h/o infection PID, TOA, ex lap LOA)

In contrast, patients with any of the following clinical features should be evaluated
for secondary causes of dysmenorrhea:
 Symptom onset at age >25
 Unilateral (nonmidline) pelvic pain (as seen in this patient's right lower
quadrant pain that radiates to the right flank)
 No systemic symptoms (eg, fatigue, nausea) during menses
 Abnormal uterine bleeding (eg, intermenstrual bleeding, postcoital spotting)

MENORRHAGIA
 Heavy prolonged menstrual bleeding; over 80 cc/ cycle
 Avg 35 ml of blood loss
 > 24 pads per day
 Estrogen increases endometrial thickness
 Progesterone matures Endometrium and withdrawal of leads to secretion
 Menstruation at regular intervals usually indicates ovulation

Abnormal Uterine Bleeding/DUB aka irregular periods indicate anovulation


Causes: fibroids, Adenomyosis, endometrial hyperplasia, endometrial polyps, cancer, pregnancy
complication
- Puberty – give Fergon, NSAIDS premarin until bleeding stops, check Von Willebrand Factor
- 16 – 40 yo think endometriosis, Adenomyosis, fibroids Tx: EMB, OCPs
- >40 yo think endometrial cancer TX: EMB, depo provera, D&C, TAH

Combined estrogen/progestin oral contraceptive pills


 Irregular, unscheduled bleeding
 Breast tenderness, nausea, bloating
 Amenorrhea
Adverse effects
 Hypertension
(most due to estrogen component)
 Venous thromboembolic disease
 Liver disorders (eg, hepatic adenoma)
 ↑ Serum triglycerides
 Pregnancy prevention
 Menstrual cycle regulation
Benefits
 ↓ Dysmenorrhea
 ↓ Hyperandrogenism (eg, acne, hirsutism)
 ↓ Risk of ovarian & endometrial cancer (if ever used)
Risk augmentation
 ↑ Risk of cervical cancer (only if currently or recently used)

METRORRHAGIA: intermenstral bleeding think endometrial polyps, endometrial/cervical cancer,


pregnancy complication

POLYMENORRHEA: cycles <21 d between periods = anovulation

OLIGOMENORRHEA: >35 d apart = disruption of pit/Gonadal axis, pregnancy

DUB: abnormal uterine bleeding in absence of organic causes

OVULATORY DUB:
 Early spotting – estrogen no increasing fast enough
 Mid spotting – estrogen drop off at ovulation
 Late spotting – Progesterone def
 TX: NSAIDS dec blood loss by 20-50%

POST MENOPAUSAL BLEEDING


- >12 months after menopause
- lower/upper genital tract
Mech: exogenous hormones
Non gyn causes: rectal bleeding, prolapse, fissures, tumors vaginal atrophy, CA (endometrial and
cervical), endometrial
Hyperplasia, Polyps
DX: inspection on PE, pap, rectal, EMB, HSG, H/H, U/S
TX: ref all gi problems, surgery, estrogen replacement, bx all lesions

HIRSUITISM / VIRILISM
Diagnosis/ Work up: assess body hair systematically
Free testosterone- ovary produces the most testosterone
DHEAS- adrenal produces the most DHEAS- screens for adrenal tumors
17 hydroxy progesterone- congenital adrenal hyperplasia
Hair type: Villus hairs – cover entire body
Terminal hairs – thick = Axillary, pubic, 5 reductase converts testosterone to dihydrotestosterone
to stimulate terminal hair development
 Hirsuitism – increase of terminal hairs esp on face, chest back, diamond shaped
escutcheon (male) increase 5 reductase
 Virilism – male features, deepening of voice, balding, increase muscle mass,
clitormegaly, breast atrophy, male body habitus
Causes: Adrenal tumor, ovarian tumor, PCO
Cushing’s syndrome: increase ACTH, cortisol
Congenital Adrenal Hyperplasia – 21 and 11 hydroxylase def

Polycystic Ovarian Syndrome: This is a syndrome which can include numerous ovarian cysts,
but really is more than that. It includes …
 Insulin Resistance: diagnosed by Fasting Glucose/ Insulin ratio <4.5 Tx: Metformin
 Hirsuitism: from hyperandrogenemia
 Anovulation: irregular, heavy periods; if desires fertility treat with metformin and clomid
 FSH : LH ratio is over 2.5:1

INFERTILITY: inability to achieve pregnancy after 12 months of unprotected intercourse, 20% of


population
- Idiopathic- 10%
- Male and Female- 10%
- Female Causes – 40%
Ovulatory – Anovulation, endocrine, PCO, premature ovarian failure
TX: ovulation induction
70% success
Clomid: antiestrogen that results in increased FSH, more mature follicies and
ovulation se: hot flashes, emotional liability, depression and mult gestations
Pergonal: purified FSH/LH HMG IM injection in follicular phase
85 – 90% effective
IVF, GIFT, ZIFT: ovulation induction, harvest oocytes add sperm fertilize place in
uterus.
Tubal: adhesions, endometriosis, PID, salpingitis
TX: tubal reconstruction
Peritoneal: endometriosis, adhesions, PID
Uterine: asherman’s, fibroids
TX: myomectomy
Luteal Phase Defect TX: progesterone during and after conception
Male Causes: 40%
TX: for all intrauterine insemination
MEDS that affect sperm analysis: cimetidine, colchicines, sulfasalazine, allopurinol,
erythromycin, steroids, tetracycline
Cyptorcidism
Varicocele
Epidydimitis
Prostatitis

Work Up:
Sperm count- must be done first
TSH, Prolactin
HSG-hysterosalpingogram- assesses patency of tubes and diagnoses intrauterine defects
Post Coital test- looks at quality of mucus and sperm, done D#12-14
BBT- temperature curve- spike predictive of ovulation
Progesterone level on day 21- assess ovulation
Diagnostic Scope- looks for endometriosis
Treatment: IRREGULAR PERIODS + Heavy bleeding Clomiphene ER antagonist ↑GnRH
OVULATION!

Pulsatile leuprolide: ↑ GnRHAI


CHANGES IN VULVA

Lichen Sclerosis – thin skin, hyalinized collagen tx: clobetasol (a high potency steroid)

Extramammary Paget’s – intraepithelial neoplasia of the skin


>60 yrs w/vulvar purities
pale atypical cells with mitotic figure
20% have adeno ca underneath
SX: pruitus unrelieved by antifungals
DX: biopsy
TX: wide local excision, Colpo
Assoc with other cancers: gi, breast, cvx c/w chronic inflammatory changes
Scar yields red velvet and white plaques on labia
Infranodal spread likely to be fatal

VIN I II III : VULVAR INTRAEPITHELIAL NEOPLASIA: dysplasia of the vulva


-atypia, thickened skin
-degree proportioned to # of mitotic fig
-can see squamouspearls
-postmenopausal late 50-60s
-correlated with HPV 80 – 90%
-diffused focal raised, flat, white, red, brown, black
SX: Vulvodynia, pruitus
TX:excision with scalpel or laser, f/u Colpo q 3 mo until disease free then q 6 mo

VULVAR CA – 5% gyn malignancy


-associated with DM, HTN, obesity vulvardystrophies
SX: Vulvodynia, purities, mass erythemia
DX: bx : see epidermoid 90% of cases, melanoma 5- 10%, basal 2-3%, cauliflower hard indurated
STAGING: I <2cm in size, no nodes, no mets
Ia <1mm
Ib >1mm
II >2cm, no nodes, no mets but can progress to perineum, urethra and anus
III unilateral nodes with any size
IV bilateral nodes
TX: based on stage, from wide local excision to vulvectomy to radical vulvectomy/lymph node
dissection

VAGINAL CA
-women in their 50’s
-DES exposure in utero resulting in clear cell adenocarcinoma
-asymptomatic for the most part but may have d/c, bleeding, purities
-TX: pap – Colpo – pathologic dx
ABNORMAL PAP SMEAR
-false negative pap 40-50%
 “benign cellular changes” : think infection so wet prep, cultures
 koilocytosis: pathologic description associated with HPV
 “ASCUS”: Atypical Squamous Cell Hyperplasia of Undetermined Significance:
o 5% hide underlying severe lesions
o repeat pap in 3 months, Colposcopy if 2 ASCUSs
o consider HPV typing
 “LGSIL”: Low Grade Squamous Intraepithelial Lesion: Tx: Colposcopy
 “HGSIL” : High Grade Squamous Intraepithelial Lesion: Tx: Colposcopy

Colposcopy: magnifies region of cervix after stained with acetic acid. Areas of dysplasia stain
WHITE (aceto white focal lesion) and are biopsied. An endocervical curettage is also done.

Treatment of dysplasia is based on the biopsy and ECC result. As a general rule…
 Mild dysplasia: observation, cryotherapy
 Moderate dyplasia: cryotheraphy or LEEP (loop electrosurgical excision procedure)
 Severe dysplasia: LEEP or Cold Knife Conization
 If ECC has dysplasia: CKC or LEEP

 4 indications for CKC:


o Microinvasion on biopsy
o ECC with dysplasia
o Pap colpo discrepancy: If the pap smear does not correlate with the biopsy
results: ie. HGSIL with normal biopsy results, you may have missed something
and need to do a CKC
o Inadequate colpo: means that there is a lesion extending into the os or that you
could not visualize the whole lesion on colpo- there may be something more
extensive there

CERVICAL CANCER
Most cancer occurs in transformation zone
Koilocyte: has viral particle
HPV oncogenic 33, 35, 52,16, 18 ordinary wart 6,11
SX: vaginal bleeding, d/c, pelvic pain, growth on cervix may palpate/see mass on exam
Classic presentation: post coital bleeding, pelvic pain/pressure, abnormal vaginal
bleeding rectal/bladder sx

Types: Squamous large cell, keratinizing, non-keratinizing, small cell (worse prog)
Adenocarcinoma
Mixed carcinoma
Glassy cell – occurs in pregnant women usually fatal

RF: tobacco # of sex partners, age of onset of sex, # STDs, HIV (cervical CA an AIDS defining
illness)

Staging – based on microinvasion so must do a cone : staged CLINICALLY


O carcinoma in situ
I contained to cervix
II carcinoma beyond cervix, no sidewall
II pelvic sidewall, hydronephrosis
IV extends beyond pelvis

TX: Ia= cone biopsy; hysterectomy 100% cure


Ib/IIa = radiation, radical hysterectomy ( takes uterus, cervix, parametrium, LN)
IIb/III/IV = extensive radiation,chemo
OVARIAN TUMORS
RF: family hx, uninterrupted ovulation, nulitips, low fertility, delayed childbearing, late onset
menopause (OCs have protective effect)
SX: asymptomatic until advanced stages, urinary frequency, dysuria, pelvic pressure, ascites, - -
Types:

Nonneoplastic: only operate if postmenopausal or if they’re over 8 cm


o Follicle cyst
o Corpus luteum Hematoma
o PCO
o Theca lutein cysts: assn with HCG and LH
o Endometrioma
o Para ovarian cysts (mullerian)

Epithelial (80%): shows a7-cm right ovarian mass with solid components, thick
septations, and a moderate amount of peritoneal fluid.
o Serous cystadenoma: papillary cystic malignant bilateral, psammonma bodies
o Endometroid: solid
o Mucinous: cystic
o Clear cell: associated with Hobnail Cells on path, assn with DES
o Brunner: look like transitional epithelium: Walthard Nests 99% benign
o SUET: solid undiff

 Acute: shortness of breath, obstipation/constipation with vomiting, abdominal distension


entation  Subacute: pelvic/abdominal pain, bloating, early satiety
 Asymptomatic adnexal mass
Laboratory
 ↑ CA-125
findings
Ultrasound  Solid mass
 Thick septations
findings
 Ascites
Management  Exploratory laparotomy

Germ Cell
o Dysgerminoma: younger people, solid radiosensitive, lymphocytic infiltrate
o Teratoma: ectoderm endoderm mesoderm, Rotikansky’s protuberance, complications:
medical: struma ovarii, autoimmune hemolytic anemia, carcinoid
surgery: torsion, acute abdomen
o Primary choriocarcinoma of the ovary false, + UPT, increased HCG
o Yolk Sac Tumor/Endodermal Sinus: +AFP/LDH, +Schuller Duval Bodies
o Mixed germ cell: HCG, AFP, LDH, CA 125

Stromal
-older women (50-80)
-Sex cords hormone production
o Fibroma: Meig’s syndrome: ovarian tumor, r hydrothorax, ascites
o Granulosa Theca – feminizing, late recurrence, Call Exner Bodies, produce large
amounts of estrogen ENDOMETRIAL CANCER get biopsy!
o AUB + als a 10-cm, complex left ovarian mass and an irregular endometrial
stripe.
o DO ENDOMETRIAL BIOPSY NEXT
Granulosa cell tumor

 Sex cord–stromal tumor


Pathogenesis  ↑ Estradiol

 ↑ Inhibin
 Complex ovarian mass
 Juvenile subtype
o Precocious puberty
Clinical features  Adult subtype
o Breast tenderness
o Abnormal uterine bleeding

o Postmenopausal bleeding
Histopathology  Call-Exner bodies (cells in rosette pattern)
 Endometrial biopsy (endometrial cancer)
 Surgery (tumor staging)
Management

o enderness (caused by estrogen-stimulated breast tissue proliferation) and


abdominal bloating/pain (eg, mass effect, ascites).  Granulosa cell tumors
secrete high levels of estradiol that cause uncontrolled endometrial proliferation;
therefore, patients often have abnormal uterine bleeding (eg, irregular menses)
due to the subsequent development of endometrial hyperplasia/cancer.
o
Sertoli Leidig – masculinizing, secrete testosterone, NORMAL DHEA, Crystaloids of Reinke
secrete androgens vs.  In contrast, elevated DHEAS and normal testosterone levels are
typically due to an adrenal tumor.
o
o A 22-year-old nulligravid woman comes to the office for worsening acne
on her face and body over the last 6 months.  She initially attributed the
acne to her diet and has been eating healthier to improve the acne and
lose weight.  The patient has lost 6.8 kg (15 lb) in the last 3 months, but
the acne has not improved.  She has no chronic medical conditions and
has had no surgeries.  The patient does not use tobacco, alcohol, or illicit
drugs.  Her father has type 2 diabetes mellitus and her mother has
hyperlipidemia.  Blood pressure is 138/86 mm Hg and pulse is 72/min. 
BMI is 42 kg/m .  Coarse hair is noted on the chin and upper
2

lip.  Nodulocystic acne is present on the face, chest, and back.  Pelvic


examination shows an enlarged clitoris but otherwise normal external
genitalia.  Bimanual examination is limited by body habitus.  Laboratory
results reveal elevated total testosterone and normal
dehydroepiandrosterone sulfate.  Which of the following is the most likely
diagnosis in this patient?
o
o Gynandroblastoma- components of male and female
o A hysterosalpingogram is used to evaluate the uterine cavity and fallopian tubes
in patients with infertility (eg, hydrosalpinx) or uterine anomalies (eg, septate
uterus). 

Other
o Hilar Cell: hillus, androgenic, small
o Krukenberg: GI metastasis
bilateral enlarged solid ovaries
signet ring cell associated with mucus
assn with gastric cancer

Ovarian Cancer Staging:


I - growth to one/both ovaries
II – with extension to pelvic structures
III – peritoneum
IV - distant mets
Adjuvant Chemo: cisplatin and taxol
XRT in II/III
Follow CA125 because increased in 80%

CA OF FALLOPIAN TUBES
-adeno CA from mucosa
-disease progresses like ovarian CA
-peritoneal spread
-ascites
-bilateral in 10-20% results from mets often
-primary in very rare
-asymptomatic but may have vague lower abdominal pain and discharge
TX: TAH/BSO cisplatin, cyclophosphomide XRT

TROPHOBLASTIC DISEASE
Moles
Complete:
-<20 yrs or >40 yrs, 80% of molar pregnancies
-Complete 46xx (both x from sperm)
-worse b/c can transform into malignant- 20 % malignant
-no baby parts
Incomplete: Triploid (usually XXY)
-May have baby parts

SX: early abnormal bleeding


-Large for dates
-+/- grape tissue
-bilateral enlarged ovaries
-increased in Asians 8/1000
-early toxemia
-threatened AB
-hyperemesis, hyperthyroid, HTN
RF: maternal age, h/o hydatidiform mole, recurrent SAB, low social economic status, poor
nutrition
TX: dilation and curettage, consider hysterectomy
F/U: monitor HCG for one year, contraception for one year (b/c don’t want to confuse rising HCG
titers of a new pregnancy with those from molar pregnancy), pelvic exams q 2 wks until uterus
clear
Chemo: if increased HCG at 6 months, lung or other mets, recurrence

Hydatidiform mole

 Abnormal vaginal bleeding ± hydropic tissue


 Uterine enlargement > gestational age
 Abnormally elevated β-hCG levels
Clinical presentation  Theca lutein ovarian cysts
 Hyperemesis gravidarum
 Preeclampsia with severe features  

 Hyperthyroidism
Risk factors  Extremes of maternal age
 History of hydatidiform mole
 "Snowstorm" appearance on ultrasound
Diagnosis  Quantitative serum β-hCG

 Histologic evaluation of uterine contents


 Dilation & suction curettage
Management  Serial serum β-hCG post evacuation

 Contraception for 6 months

CHORIOCARCINOMA: malignanancies in assn with pregnancy


-majority follow trophoblastic moles, but can follow normal pregnancy also
-1/20,000 pregnancies
RF: as above (A) women mating with (O) men
SX: abnormal bleeding after any pregnancy
TX:
Chemotherapy
1. MTX
2. Etoposide/actinomycin D/MTX
3. Cyclophosphamide/Vincristine
D&C

CONTRACEPTION
Rhythm
Fertility awareness/abstinences
55-80% effective
ovulation assment = BBT
menstrual cycle tracking
cervical mucus exam

Coitus Interuptus
Withdrawal before ejaculation
15-25% failure

Lactational Amenorrhea
Nursing delays ovulation by hypothalamic suppression
Max of 6 months
50% ovulate by 6-12 months
15-55% get pregnant while nursing

Barrier
Male and female condom, diaphragm, cervical cap sponge, spermacide

IUD
Spermicidal inflammatory response/ inhibition of implantation
Used when OCPs contraindicated
Patient is a low STD risk
Contraindicated in pregnancy, abnormal vaginal bleeding, infection
Relative contraindication: nullip, prior ectopic, h/o STD, mod/sev dysmenorrhea
Failure rate <2%
Norplant: not sold anymore for monetary reasons only
Sustained release- 5 years
0.2% failure
not many side effects b/c no estrogen only progesterone
six flexible rods (36mg progesterone) SQ upper arm
side effects: Irregular vaginal bleeding, HA, wt change, mood changes

Deproprovera
Medoxyprogesterone acetate
IM slow release of over 3 months
.3% failure rate
side effects: irregular menstrual bleeding, depression, weight gain
>70% get irregular menses, eventually have amenorrhea

Vasectomy
Ligation of the vas deferens
<1% failure rate
must use condom for 4-6 wks until azospermia confirmed on semen analysis
70% reanastomose resulting in pregnancy 18-60%
50% make anti-sperm antibodies

Tubal Sterilization
Most used method of birth control
4% failure rate
No side effects
Permanent although 1% seek reversal which is successful in 41-84%
1/1,500 risk of ectopic
4/100,000 mortality rate

Oral Contraceptive Pills:


MECH: Pulsatile release of FSH and LH suppresses ovulation
Change in cervical in cervical mucus
Change in Endometrium

TYPES:
Monophasic – fixed dose of estrogen and progesterone
Multphasic varies progesterone dose each week and lower overall estrogen/prog
Progesterone progestin only not as effective as combination OCPs

COMPLICATIONS:
Thromboembolism ( do not give in women with family history of DVT or PE), PE, CVA,
MI, HTN

MEDS that Decrease Efficacy of OCPS:


PCN, tetracycline, rifampin, ibuprofen, dilantin, barbiturates, sulfonamide

OCP decrease the efficacy of folates,anticoagulants, insulin, methyldopa, phenothiazine

Benefits of OCP:
Decrease ovarian/endometrial ca (BY 50%!!!), ectopic, anemia, pid, cysts, benign breast
dz, osteoporosis.
CONTRAINDICATIONS
Vulvar cancer

 Persistent HPV infection


Etiology
 Chronic inflammation
 Tobacco use
 Vulvar lichen sclerosus
Risk factors  Immunodeficiency
 Prior cervical cancer

 Vulvar/cervical intraepithelial neoplasia


 Vulvar pruritus
Clinical features  Vulvar plaque/ulcer

 Abnormal bleeding
Diagnosis  Biopsy

HPV = human papillomavirus.

Absolute contraindications
to combined hormonal contraceptives

 Migraine with aura


 >15 cigarettes/day PLUS age >35
 Hypertension >160/100 mm Hg
 Heart disease
 Diabetes mellitus with end-organ damage
 History of thromboembolic disease
 Antiphospholipid-antibody syndrome
 History of stroke
 Breast cancer
 Cirrhosis & liver cancer
 Major surgery with prolonged immobilization

 Use <3 weeks postpartum


THERAPUTIC AB
 25% of pregnancies end in therapeutic ab
 Risk of death < 1/100,000 (anesthesia)
 Vaginal evacuation – suction curettage, D & C/E
 Induction of labor
 Medical TX :
o Antiprogestin agent (RU-486 – mifepristone : blocks effects of progesterone) 1st
½ of 1st trimester.
o Post coital pill – high doses of estrogen that either suppresses ovulation or
accelerates ovum thru tube so no fertilization se: N/V
 2nd Term
 Congenital anomalies
 Vaginal prostaglandin
 D&E
 Induction of labor w/ hypertonic solution (saline, urea, PGF, PGE vaginal suppositories)

Ulcerative Cholitis: primigravida patient with hematochezia, abdominal pain,


and tenesmus (eg, fecal urgency followed by straining and inability to defecate) most
likely has ulcerative colitis (UC).  Pregnancy is a high-risk period for patients with UC
due in part to placental cytokines that worsen colonic inflammation.  Maternal
complications include worsening UC disease activity, which can lead to severe
hematochezia, anemia, and, in rare cases, toxic megacolon.  Fetal risks include preterm
delivery and small for gestational age.  In addition, fertility rates are lower in those
with active UC.

MGMT: Most medications used in the management of UC (eg, mesalamine,


sulfasalazine, tumor necrosis factor-alpha inhibitors) are considered safe
for continuation throughout pregnancy.  These medications can also be continued
during breastfeeding, with the exception of sulfasalazine, which is not recommended due
to insufficient safety data.

Ddx
Gastrointestinal arteriovenous malformation (AVM) usually causes occult gastrointestinal
bleeding and presents with iron deficiency anemia; hematochezia is less common.  In
addition, this patient lacks risk factors for AVM (eg, chronic kidney disease, aortic
stenosis, and von Willebrand disease).
(Choice B)  Diverticular bleeding typically presents with hematochezia in the absence of
abdominal pain.  It is generally diagnosed in patients age >60; pregnancy is not
considered a risk factor.
(Choice C)  Familial adenomatous polyposis (FAP) is an autosomal dominant condition
characterized by hundreds of colonic adenomatous polyps.  Affected patients universally
develop colon cancer, often at a young age.  This patient does not have any significant
family history, making FAP unlikely.
(Choice D)  Internal hemorrhoids are very common in pregnancy and present with
streaky, bloody stools in the setting of constipation and rectal itching.  Internal
hemorrhoids would not explain tenesmus and fetal growth restriction.
(Choice E)  Shigellosis presents acutely with fevers, abdominal pain, tenesmus, and
bloody diarrhea for up to 7 days.  It is unlikely in this patient, who has had symptoms for
4 weeks.

Patau: polydact

Birth Injuries
ERBS vs klumkes

You might also like